You are on page 1of 33

Clinical Anatomy: An Illustrated Review with Questions and Explanations by Richard Snell 4th Ed.

1. The broad ligament contains all of the following except *B. A transverse foramen in each vertebra
which? C. A small circular vertebral foramen
A. The round ligament of the ovary D. Progressively shorter transverse processes from T10-12
B. The uterine artery E. Thoracic articular processes set on an arc to permit rotation
C. The round ligament of the uterus
D. The uterine tubes 9. Which of the following terms correctly applies to abnormal
*E. The ureters. curvatures of the vertebral column?
A. Sacralization
2. Malignant tumors of the trigone of the bladder spread B. Lumbarization
(metastasize) to which of the following lymph *C. Kyphosis
nodes? D. Osteoporosis
A. Lumbar E. Osteomalacia
B. Sacral
C. External iliac only 10. Lordosis is characterized by an increased curve of the
*D. External and internal iliac vertebral column that is:
E. Superficial inguinal. A. Convex posteriorly
*B. Convex anteriorly
3. Which of these statements correctly describes the obturator C. Convex to the side
nerve? D. Concave laterally
A. It arises from the sacral plexus E. Concave anteriorly
*B. It passes along the lateral pelvic wall
C. It supplies abductor muscles of the thigh 11. A herniated or prolapsed disc usually occurs in which of the
D. It arises from posterior rami of sacral nerves following directions?
E. It gives off pelvic splanchnic nerves A. Anterior
B. Posterior
4. The broad, yellow, elastic bands that join the laminae of *C. Posterolateral
adjacent vertebral arches are known as the: D. Anterolateral
A. Interspinous ligaments E. Inferiorly
B. Supraspinous ligaments
C. Ligamentum nuchae 12. In severe neck flexion injuries, which of the following
D. Intertransverse ligaments ligaments usually is torn?
*E. Ligamenta flava A. Anterior longitudinal
*B. Posterior longitudinal
5. Which of the following statements correctly applies to the C. Apical
medial column of the erector spinae D. Ligamentum nuchae
muscle? E. Ligamentum flavum
A. It is known as the longissimus muscle
*B. It arises from spinous processes 13. The subarachnoid space ends inferiorly at the level of the:
C. It inserts into transverse processes A. L5 vertebra
D. It is a flexor of the vertebral column B. L3 vertebra
E. It is a superficial muscle *C. S2-3 vertebrae
D. T12 vertebra
E. L1 vertebra.
6. Which of the following muscles is concerned with the
maintenance of posture and movements of the 14. In adults, the spinal cord usually ends inferiorly at the:
vertebral column? A. Lower border of S2
A. Serratus posterior inferior B. Upper border of S1
B. Trapezius C. Lower border of S4
C. Latissimus dorsi D. Upper border of coccyx
D. Levator scapulae *E. Lower border of L1.
*E. Longissimus
15. The least serious congenital abnormality involving the neural
7. The erector spinae muscle is innervated by the: arch and the neural tube is:
A. Axillary nerve A. Rachischisis
*B. Dorsal rami of spinal nerves B. Meningohydroencephalocele
C. Accessory nerve C. Meningomyelocele
D. Branches from the posterior cord of brachial plexus *D. Spina bifida occulta
E. Dorsal scapular nerve E. Meningoencephalocele.

8. Characteristics of thoracic vertebrae include all of the 16. To perform a lumbar puncture (spinal tap) in an adult, the
following except: needle is introduced between the spinous
A. Long vertical spinous processes of T5, T6, T7, and T8 processes of:
Clinical Anatomy: An Illustrated Review with Questions and Explanations by Richard Snell 4th Ed.

*A. L4 and 5 A. The intervertebral discs make up approximately one fourth the
B. L2 and 3 length of the vertebral column
C. S1 and L2 B. In old age, atrophy of the intervertebral discs tends to produce
D. T12 and L1 a continuous posterior convexity of the
E. S3 and L4. vertebral column
C. During pregnancy, the weight of the developing fetus
17. How many articulations form the Lisfrank’s joint: increases the posterior lumbar concavity of the
*A. 2 vertebral column
B. 3 D. The odontoid process of the axis represents developmentally
C. 4 the body of the atlas
D. 5 *E. There are seven cervical spinal nerves and eight cervical
E. 6. vertebrae.

18. Lymph from the skin of the back in the region of the spinous 24. The following statements concerning the vertebral column
process of the tenth thoracic vertebra are correct except which?
drains into the: A. The atlantoaxial joints permit rotation of the atlas with the
A. Posterior mediastinal lymph nodes head on the axis.
B. Superficial inguinal lymph nodes B. The posterior ramus of the first cervical spinal nerve and its
*C. Axillary lymph nodes continuation, the great occipital nerve,
D. Sacral lymph nodes supplies the skin over the back of the scalp
E. None of the these. *C. When an individual is in the standing position, the line of
gravity passes anterior to the cervical part
19. The _________ passes between the sixth and seventh cervical of the vertebral column and posterior to the thoracic and lumbar
vertebrae: regions of the column
A. Vertebral artery D. The tip of the spine of a thoracic vertebra lies directly behind
*B. Seventh cervical spinal nerve the vertebral body of the vertebra below
C. Sixth cervical spinal nerve E. The intervertebral disc is innervated by a recurrent branch of
D. Vertebral vein spinal nerve that enters the vertebral
E. Eight cervical spinal nerve. canal through the intervertebral foramen

20. The cauda equina is made up of the: 25. When performing a lumbar puncture (spinal tap), the
A. Spinal nerves of S1-3 following structures are pierced by the needle
B. Anterior rami of spinal nerves L2 and 3 except which?
C. Posterior rami of spinal nerves L1 to cocc 1 *A. The posterior longitudinal ligament
*D. Anterior and the posterior nerve roots of the spinal nerves B. The supraspinous ligament
below the first lumbar segment of the C. The arachnoid mater
spinal cord D. The
E. Spinal nerves of L1-5.
26. All the following characteristics are present in a lumbar
21. A herniated disc that causes sensory changes in a specific vertebra except which?
dermatome is pressing on: A. There is a massive kidney-shaped body
A. An anterior primary ramus *B. The transverse processes are short and thick
B. An anterior gray horn of the spinal cord C. The spinous processes are short, flat, and quadrangular in
C. An anterior root shape
D. A posterior primary ramus D. The transverse processes have no foramen
*E. A spinal nerve or a posterior root. E. The articular surfaces of the superior articular processes face
medially
22. Which of the following statements regarding the blood
supply to the spinal cord is true? 27. The following statements regarding the internal vertebral
*A. The posterior spinal arteries supply the posterior third of the venous plexus are correct except which?
spinal cord A. It drains blood from the vertebral bodies
B. The anterior spinal arteries are two in number and run down B. It permits malignant cells from the prostate to metastasize to
the anterior surface of the spinal cord the skull
close to the anterior nerve roots *C. It does not possess competent velves
C. The veins of the spinal cord drain into the external vertebral D. The venous flow is indirectly influenced by changes in the
venous plexus intra-abdominal pressure
D. The anterior and the posterior spinal arteries do not E. It does not communicate with veins within the thorax
anastomose with the radicular arteries
E. The spinal cord has a profuse blood supply. 28. The following statements regarding herniation of an
intervertebral disc are correct except which?
23. The following statements concerning the vertebral column A. It generally herniates posteriorly
are correct except which?
Clinical Anatomy: An Illustrated Review with Questions and Explanations by Richard Snell 4th Ed.

B. In the lumbar region, it usually affects the spinal nerve whose


number corresponds to the vertebra
below 35. Which of the following structures contributes to the
C. It is a portion of the nucleus pulposus that actually herniates formation of the lumbar triangle?
D. *A contributing factor to the herniation may be excessive A. Rectus sheath
compression of the posterior region of the *B. Crest of the ilium
disc C. Ischial tuberosity
E. It may press on the spinal cord. D. Inguinal ligament
E. Serratus posterior.

29. The following statements regarding an intervertebral disc are 36. The psoas major muscle inserts on to which of the following
correct except which? structures?
A. During aging, the fluid within the nucleus pulposus is A. Greater trochanter
diminished and the amount of fibrocartilage is B. Anterior superior iliac spine
increased C. Crest of the ilium
*B. The atlantoaxial joint possesses a small disc *D. Lesser trochanter
C. The discs play a major role in development of the curvatures E. Ischial spine.
of the vertebral column
D. The discs are thickest in the lumbar region 37. Which of the following structures is located in the renal
E. The discs are innervated by adjacent spinal nerves column?
*A. Interlobular arteries
30. The weakness of the left deltoid and biceps brachii muscles B. Collecting tubule
could have resulted from pressure on C. Arcuate arteries
which nerve roots? D. Interlobar arteries
A. The posterior nerve roots of C5and 6 E. Minor calyx.
*B. The anterior nerve roots of C5 and 6
C. The anterior nerve roots of C7 and 8 38. Which of the following vessels and organs are situated in
D. The anterior nerve roots of C8 and T1 retroperitoneal space?
E. None of these. *A. Kidney, suprarenal gland, ureter, abdominal part of aorta,
inferior vena cava
31. The following structures form the boundaries of B. Aorta, kidney, ureter
intervertebral foramen except which? C. Suprarenal gland, kidney, inferior vena cava
A. Intervertebral discs D. Kidney, suprarenal gland, ureter
B. Bodies of the cervical vertebrae E. Kidney, suprarenal gland, renal arteries.
*C. The posterior longitudinal ligament
D. Articular processes of the vertebrae 39. What is nephrostomy?
E. Pedicles of the vertebrae. *A. Renal fistula
B. Incision of the kidney
C. Cutting of renal pelvis
32. The right suprarenal vein drains into which of the following D. Decapsulation of the kidney
veins? E. Extraction of the kidney.
A. Right renal
*B. Inferior mesenteric 40. What is the name of operation of kidney fixation during
C. Superior mesenteric nephroptosis?
D. Portal *A. Nephropexy
E. Inferior vena cava B. Nephrotomy
C. Neprostomy
33. Which of the following arteries contributes to the blood D. Nephrectomy
supply of the pancreas? E. Pyelotomy.
A. Left gastroepiploic
B. Inferior mesenteric 41. What is the name of operation of renal calculus extraction
C. Proper hepatic from the renal pelvis?
*D. Splenic *A. Pyelotomy
E. Left colic. B. Nephrotomy
C. Nephrectomy
34. The suprarenal gland receives its blood supply from which of D. Nephropexy
the following arteries? E. Neprostomy.
*A. Renal
B. Gonadal 42. The perineum is bordered by the:
C. Superior mesenteric A. Tuber ischiadicus
D. Inferior mesenteric B. Lig. sacrotuberale
E. Superior phrenic. C. Coccyx
Clinical Anatomy: An Illustrated Review with Questions and Explanations by Richard Snell 4th Ed.

D. Inferior branches of the pubic bone *A. Iliofemoral ligament


*E. All of the above. B. Ischiofemoral ligament
C. Pubofemoral ligament
43. Within the femoral triangle, the femoral vein and artery are D. Quadriceps muscle
encased in a fascial femoral sheath. The E. Adductor magnus muscle.
medial part of this sheath contains a small, vertical gap - the
femoral ring, through which herniation 51. Flexion of the hip joint (with the knee flexed) is limited by
may occur. The boundaries of the femoral ring include all the the:Hamstring muscle
following except the: A. Iliofemoral ligament
*A. Femoral nerve B. Adductor magnus muscle
B. Femoral vein C. Pubofemoral ligament
C. Inguinal ligament *D. Anterior abdominal wall
D. Lacunar ligament E. Adductor magnus muscle
E. Pectineal ligament (of Cooper
52. The long head of the biceps femoris muscle is innervated by
44. The superficial epigastric artery arises from which of the the;
following arteries? A. Obturator nerve
*A. Femoral *B. Tibial portion of the sciatic nerve
B. Internal iliac C. Femoral nerve
C. External iliac D. Common peroneal nerve
D. Inferior epigastric E. Sural nerve.
E. Superior epigastric
53. The gracilis muscle is innervated by the:
45. The superficial circumflex iliac artery arises from which of A. Femoral nerve
the following arteries? B. Common peroneal nerve
A. External iliac C. Sural nerve
B. Internal iliac *D. Obturator nerve
C. Superior epigastric E. Tibial portion of the sciatic nerve.
D. Inferior epigastric
*E. Femoral artery 54. The gluteus maximus muscle is innervation by the:
*A. Inferior gluteal nerve
46. Branches of the sacral plexus include which of these nerves? B. Nerve to quadratus femoris
A. Obturator C. Obturator nerve
B. Femoral D. Superior gluteal nerve
*C. Sciatic E. Nerve to obturator internus.
D. Iliohypogastric
E. Genitofemoral 55. The sartorius muscle is innervated by the:
A. Obturator nerve
47. Femoral hernia descend into the thigh behind which of the *B. Femoral nerve
following ligaments? C. Nerve to vastus medialis
A. Lacunar D. Superior gluteal nerve
B. Iliopectineal E. Lateral cutaneous nerve of the thigh.
C. Reflected inguinal
D. Falx inguinalis 56. The hamstring portion of the adductor magnus muscle is
*E. Inguinal innervated by the:
A. Femoral nerve
48. Flexion of the hip joint (with the knee extended) is limited by B. Nerve to the pectineus muscle.
the: C. Common peroneal nerve
A. Iliofemoral ligament *D. Tibial portion of the sciatic nerve
B. Anterior abdominal wall E. Obturator nerve
C. Ischiofemoral ligament
D. Pubofemoral ligament 57. The adductor longus muscle is innervated by the:
*E. None of these. A. Femoral nerve
B. Common peroneal nerve
49. Abduction of the hip joint is limited by the: C. Tibial portion of the sciatic nerve
A. Pectineus muscle *D. Anterior division of the obturator nerve
B. Iliofemoral ligament E. Posterior division of the obturator nerve.
C. Ischiofemoral ligament
*D. Pubofemoral ligament 58. A malignant melanoma (cancer) of the skin covering the
E. Ligament of the head of the femur. buttock is likely to spread via lymphatics to
the:
50. Extension of the hip joint is limited by the: *A. Horizontal group of superficial inguinal nodes
Clinical Anatomy: An Illustrated Review with Questions and Explanations by Richard Snell 4th Ed.

B. Vertical group of superficial inguinal nodes A. The medial plantar artery


C. Posterior axillary nodes B. The peroneal artery
D. Popliteal nodes C. The posterior tibial artery
E. Internal iliac nodes. D. The lateral plantar artery
*E. All of these
59. Lymph from the nail bed of the big toe drains into the:
A. Horizontal group of superficial inguinal nodes 67. In children, the chief arterial supply to the head of the femur
B. Presacral nodes is derived from which of the following
C. Popliteal nodes artery or arteries?
D. Internal iliac nodes *A. The obturator artery
*E. Vertical group of superficial inguinal nodes. B. The internal pudendal artery
C. Branches from the medial and lateral circumflex femoral
60. The lateral meniscus of the knee joint: arteries
A. Has a thick inner border D. The deep circumflex iliac artery
B. Is strongly attached around its circumference to the fibia E. The superficial circumflex iliac artery.
C. Is more frequently torn than the medial meniscus
D. Is strongly attached to the lateral collateral ligament 68. Which statement is correct concerning the femoral ring?
*E. Is attached by its anterior horn to the tibia in front of the A. It is the opening in the deep fascia of the thigh for the great
intercondylar eminence. saphenous vein
B. It is the opening in the adductor magnus muscle for the
61. Lymph from the skin of the lateral side of the foot drains into femoral artery
the; *C. It is the proximal opening in the femoral canal
*A. Popliteal nodes D. It is the compartment in the femoral sheath for the femoral
B. Vertical group of superficial inguinal nodes artery
C. Internal iliac nodes E. It is the compartment in the femoral sheath for the femoral
D. Horizontal group of superficial inguinal nodes nerve.
E. Subsartorial nodes.
69. To lift the left foot off the ground while walking, which of
62. Lymph from the skin of the medial side of the knee drains the following muscles play an important
into the: role?
A. Popliteal nodes A. The left gluteus medius muscle
*B. Vertical group of superficial inguinal nodes B. The left gluteus maximus muscle
C. Internal iliac nodes C. The right adductor longus muscle
D. Medial group of the horizontal superficial inguinal nodes D. The right gluteus medius muscle
E. Lateral group of the horizontal inguinal nodes. *E. None of the above

63. Lymph from the skin around the anus drains into the: 70. Rupture of the tendo calcaneus results in an inability to do
A. Internal iliac nodes what?
B. Inferior mesenteric nodes A. Dorsiflex the foot
C. Lateral group of the horizontal superficial inguinal nodes B. Evert the foot
*D. Medial group of the horizontal superficial inguinal nodes C. Invert the foot
E. Vertical group of inguinal nodes. *D. Plantar flex the foot
E. None of the above.
64. The sciatic nerve enters the gluteal region through which
foramen? 71. If the foot is permanently dorsiflexed and everted, which
A. Posterior sacral nerve might be injured?
*B. Greater sciatic A. The deep peroneal nerve
C. Anterior sacral B. The superficial peroneal nerve
D. Lesser sciatic C. The common peroneal nerve
E. Obturator. *D. The tibial nerve
E. The obturator nerve
65. The common peroneal nerve can be palpated in which region
of the knee? 72. The femoral nerve arises from which of the following
A. As it winds around the medial side of the neck of the fibula segments of the spinal cord?
B. As it passes around the medial condyle of the tibia A. L2 and 3
*C. As it winds around the lateral side of the neck of the fibula B. L4 and 5 and S1, 2, and 3
D. As it passes forward between the tibia and the fibula *C. L2, 3, and 4
E. As it crosses the lateral side of the head of the fibula. D. L1 and 2
E. L5 and S1, 2, and 3.
66. If the dorsalis pedis artery is severed just proximal to its
medial and lateral tarsal branches, blood can 73. Which of the following muscles everts the foot?
still reach the dorsum of the foot through which vessel(s)? A. The tibialis posterior muscle
Clinical Anatomy: An Illustrated Review with Questions and Explanations by Richard Snell 4th Ed.

B. The flexor hallucis longus muscle B. Lesser ischiadic muscle


*C. The peroneus longus muscle C. Inferior gemellus muscle
D. The tibialis anterior muscle D. Piriform muscle
E. The flexor digitorum longus muscle. E. Superior gemellus muscle.

74. The femoral sheath is formed by which of the following 81. All the following statements about the sartorius muscle are
layer(s) of fascia? correct except which?
A. The pectineus fascia A. It flexes the leg at the knee joint
*B. The fascia iliaca and the fascia transversalis B. It flexes the thigh at the hip joint
C. The fascia lata and the membranous layer of the superficial C. It laterally rotates the thigh at the hip joint
fascia *D. It adducts the thigh at the hip joint
D. The psoas fascia and the fatty layer of superficial fascia E. It attaches to the anterior superior iliac spine.
E. The processus vaginalis.
82. The following structures are transmitted through the lesser
75. Injury to the common peroneal nerve results in which sciatic foramen EXCEPT which?
conditions? A. The tendon of the obturator internus muscle
A. Inability to invert the foot B. The internal pudendal vessels
B. Inability to plantar flex the ankle C. The nerve to the obturator internus muscle
C. Inability to feel skin sensation on the medial side of the leg D. The pudendal nerve
D. Inability to plantar flex the big toe *E. The inferior gluteal artery.
*E. Inability to evert the foot.
83. The following statements regarding the great saphenous vein
76. After a lesion of the tibial part of the sciatic nerve, some are correct EXCEPT which?
active flexion may still be possible at the A. It arises on the dorsum of the foot
knee joint; the muscle responsible for this remaining flexion B. It enters the leg by passing anterior to the medial malleolus
include which? C. It drains into the femoral vein approximately 1,5 in (3,8 cm)
*A. The short head of the biceps femoris muscle below and lateral to the pubic tubercle
B. The gastrocnemius muscle D. It is accompanied by the saphenous nerve
C. The plantaris muscle *E. It has no communication with the deep veins of the leg.
D. The popliteus muscle
E. The long head of the biceps femoris muscle. 84. The following statements regarding the obturator nerve are
correct EXCEPT which?
77. A patient in the supine position with the hip and knee joints A. It originates from the lumbar plexus
extended is asked to abduct the lower *B. It enters the thigh immediately beneath the inguinal ligament
limb against resistance provided by the physician; this exercise C. It innervates the adductor muscles of the thigh
tests which of the following muscles? D. It divides into an anterior and a posterior division
A. The semitendinosus muscle E. It supplies the skin on the medial side of the thigh.
*B. The gluteus medius muscle
C. The pectineus muscle 85. Which of the following anatomic structures pass through the
D. The gracilis muscle suprapiriform foramen?
E. The semimembranous muscle. *A. A. glutea superior
B. M. piriformis
78. A femoral hernia has the following characteristics except C. A. glutea inferior
which? D. N. pudendus
A. It is more common in women than in men E. M. gemellus superior.
B. The swelling occurs below and lateral to the pubic tubercle
C. It descends through the femoral canal 86. Which of the following anatomic structures pass through the
*D. Its neck is related immediately laterally to the femoral artery suprapiriform foramen?
E. Its neck is related medially to the sharp edge of the lacunar A. N. ischiadicus
ligament. B. N. pudendus
*C. N. gluteus superior
79. The gastrocnemius and the soleus muscles have all the D. N. cutaneus femoris posterior
following features in common except which? E. None of these.
A. They are supplied by the tibial nerve
B. They are found in the posterior compartment of the leg 87. Which of the following anatomic structures pass through the
*C. They arise from the femoral condyles and flex the knee joint infrapiriform foramen?
D. They insert via the tendo calcaneus *A. A. pudenda interna
E. They plantar flex the ankle joint. B. A. glutea superior
C. A. glutea inferior
80. Which of the following muscles pass through the lesser D. A. obturatoria
ischiadic foramen: E. A. femoralis.
*A. Obturator internus muscle
Clinical Anatomy: An Illustrated Review with Questions and Explanations by Richard Snell 4th Ed.

88. Which of the following anatomic structures pass through the 96. Each of the following structure crosses the pelvic brim except
infrapiriform foramen? the:
A. A. glutea superior A. Ovarian artery
*B. N. ischiadicus B. Ureter
C. V. glutea superior C. Round ligament of the uterus
D. N. gluteus superior *D. Uterine artery
E. N. cutaneus femoris lateralis E. Lumbosacral trunk

89. N. ischiadicus pass between which of the following muscles 97. Which of the following structures is most important for
of the thigh: support of the female pelvic viscera?
*A. Long head of biceps of thigh and semitendinosus muscle A. The supporting fasciae
with semimembranosus muscle B. The perivascular stalk as a suspensory structure
B. External obturator muscle and internal obturator muscle C. The uterosacral ligament
C. Semimebranous muscle and semitendinous muscle *D. The levator ani muscle
D. Vastus medialis muscle and sartorius muscle E. The round ligament of the uterus
E. Adductor longus muscle and semitendinous muscle.
98. The parasympathetic preganglionic nerve fibers to the wall of
90. The biggest branch of femoral artery is: the urinary bladder have their cell
A. Superficial epigastric artery bodies in the:
B. Inferior epigastric artery A. Medulla oblongata
*C. Deep artery of thigh B. Lateral horn of all thoracic cord levels
D. External pudendal artery *C. Sacral spinal cord segments 2,3, and 4
E. Rectal artery. D. Lateral horn of the spinal cord at upper lumbar levels
E. Lateral horn of all thoracic and upper three lumbar levels of
91. The medial wall of vascular lacuna is formed by: the spinal cord
A. Femoral vein
B. Superior crural artery
*C. Lacunar ligament 99. Each of the following arteries is a direct or indirect branch of
D. Iliopectineal arch the internal iliac artery except the:
E. Pectineal ligament A. Obturator
B. Artery of the ductus deferens
92. The lateral wall of vascular lacuna is formed by: *C. Inferior epigastric
*A. Iliopectineal arch D. Umbilical
B. Lacunar ligament E. Middle rectal
C. Superior crural artery
D. Pectineal ligament 100. The internal spermatic fascia is derived from the:
E. Internal iliac artery. A. External abdominal oblique aponeurosis
B. Internal abdominal oblique aponeurosis
93. The medial wall of muscular lacuna is formed by: C. Transversus abdominis aponeurosis
A. Femoral nerve *D. Transversalis fascia
B. Lacunar ligament E. Peritoneum
C. Inguinal ligament
*D. Iliopectineal ligament 101. Each of the following characterizes the levator ani muscle
E. Pubic ligament. except:
A. It forms the principal part of the pelvic diaphragm
94. What is reamputation: B. It generally shows a separation into three parts
A. Second amputation C. It arises from the inner surface of the superior ramus of the
B. Amputation after 48 hours pubis
C. Amputation after 7-8 days D. It resists increased intraabdominal pressure
D. Final amputation E. * It separates the superficial and deep inguinal spaces
*E. Repeated amputation
102. The deep perineal space in the male contains each of the
95. The female reproductive viscera are best characterized by following except the:
which of the following statements? A. Membranous urethra
*A. The mesosalpinx contains the tubal branches of the uterine B. * Prostate gland
vessels C. Bulbourethral glands
B. The ovarian veins drain directly into the inferior vena cava D. Deep transverse perineus muscle
C. Lymph from the cervix drains into the inguinal nodes E. Sphincter urethral muscle
D. Visceral afferent nerves from the body of the uterus course
along the pelvic splanchnic nerves 103. Each of the following characterizes the pudendal canal
E. The ovarian veins drain directly into the inferior vena cava except:
A. It is formed by a split in the obturator internus fascia
Clinical Anatomy: An Illustrated Review with Questions and Explanations by Richard Snell 4th Ed.

B. It is located on the lateral wall of the ischiorectal fossa E. * Smooth muscle


C. It transmits the pudendal nerve 111. The inferior epigastric artery arises from which of the
D. It transmits the internal pudendal artery and vein following arteries?
E. * It transmits lymphatics from the upper third of the rectum A. Internal thoracic
B. * External ilia
104. The perineum receives its primary motor supply from which C. Femoral
of the following nerves? D. Obturator
A. * Pudendal E. Musculophrenic
B. Inferior gluteal
C. Superior gluteal 112. The superficial external pudendal artery emerges through
D. Posterior femoral cutaneous which of the following?
E. Ilioinguinal A. Deep inguinal ring
B. * Saphenous opening
105. Pelvic splanchnic nerves are: C. Lumbar triangle
A. * Parasympathetics from the second, third, and fourth sacral D. Arcuate line
spinal cord segments E. Rectus sheath
B. Sympathetics to the second, third, and fourth sacral nerves
C. Direct branches from the sacral sympathetic trunk 113. Among the coverings of the cord and testis, which of the
D. Postganglionic sympathetic fibers ascending to the abdomen following represents the internal abdominal oblique muscle
andthorax layer?
E. Direct branches from the lumbar plexus to the pelvic viscera A. Tunica tartos
B. External spermatic fascia
106. All of the following statements concerning the uterus are C. * Cremaster muscle and fascia
correct except: D. Internal spermatic fascia
E. Subcutaneous layer
A. The peritoneum reflects from the posterior surface of the
uterus to form the rectouterine pouch 114. The cremaster muscle is innervated by which of the
B. The normal position of the uterus is described as anteverted following nerves?
and anteflexed A. Ilioinguinal
C. The peritoneum extends from the anterior surface of the uterus B. Iliohypogastric
onto the bladder C. Femoral
D. * It receives its blood supply from a vessel that courses deep D. Subcostal
to the ureter E. * Genitofemoral
E. The uterine tubes enter the uterus inferior to the fundus of the
uterus 115. The iliolumbar artery is a branch of which of the following
arteries?
107. The crura of the clitoris or penis are attached to the: A. * Internal iliac
B. External iliac
A. Transverse perineal ligament C. Internal thoracic
B. Pubic symphysis D. Femoral
C. Ischial spines E. Inferior epigastric
D. * Ischiopubic rami
E. Ischial tuberosities 116. The remnant of the gubernaculum testis is known as which
108. The highest point of the iliac crest is at the transverse level of the following?
of which of the following structures? A. Processus vaginalis
A. Inguinal ligament B. Tunica vaginalis testis
B. Xiphisternal joint C. Inguinal canal
C. * Fourth lumbar vertebra D. * Scrotal ligament
D. Anterior iliac spine E. Genitoinguinal ligament
E. ischial tuberosity
109. Which of the following types of pelvis predominates in the 117. Which of the following structures is not located in the
male? spermatic cord?
A. Gyn'ecoid and android A. Ductus deferens
B. * Anthropoid and android B. Deferential artery
C. Platypelloid and anthropoid C. Testicular artery
D. Gynecoid and platypelloid D. Pampiniform plexus of veins
E. PlatypeUeid and android E. * Urethra
110. The tunica dartos scroti contains which of the following?
A. Fat 118. The tunica vaginalis testis is a remnant of which of the
B. Striated muscle following?
C. The iliohypogastric nerve A. Urachus
D. The inferior epigastric artery B. * Processus vaginalis
Clinical Anatomy: An Illustrated Review with Questions and Explanations by Richard Snell 4th Ed.

C. Scrotal ligament 126. These statements about pelvic splanchnic nerves are correct
D. Gubemaculum testis except for which of the following?
E. Genitoinguinal ligament A. They contain preganglionic parasympathetic fibers
B. They convey visceral afferents from the pelvic plexus to sacral
119. The pelvic splanchnic nerves provide parasympathetic fibers segments of the spinal cor
to all of the following structures except: C. They contribute to formation of the inferior hypogastric
A. Bladder plexus
B. * Right colic flexure D. * They are branches of the sciatic nerve
C. Descending colon E. They are known as nervi erigentes
D. Sigmoid colon
E. Distal one third of the transverse colon 127. In relation to autonomic plexuses of the pelvis, all these
items are correct except which one of the following?
120. Which of these statements does not describe differences A. The superior rectal plexus consists chiefly of sympathetic
between male and female pelvis accurately? fibers
A. The female pelvis is more cylindrical than the male B. The ovarian plexus consists chiefly of sympathetic fibers
B. The female sacrum is shorter and wider than the male C. Parasympathetic "components predominate in the inferior
C. The female pubic tubercles are farther apart than the male hypogastric plexus
D. * The female pubic arch makes a more acute angle than in the D. * The superior hypogastric plexus is known as the pelvic
male plexus
E. The female anterolateral pelvic wall is relatively wider than in E. Hypogastric nerves help to form the inferior hypogastric
the male plexus

121. The pelvic diaphragm is composed of all the following 128. Which of these statements does not describe correctly the
muscles except the: pelvic splanchnic nerves?
A. Coccygeus A. They are known as nervi erigentes
B. Iliococcygeus B. They are major pathways for visceral afferents from pelvic
C. * Piriformis viscera
D. Pubococcygeus C. * Afferents from the uterus follow a route through these
E. Puborectalis nerves
122. Visceral branches of the internal iliac artery include all D. Their visceral efferent fibers have cell bodies at spinal cord
except which one of the following? levels S2-4
A. Umbilical
B. Inferior vesical E. Their visceral afferents ascend to a dorsal root ganglion cell
C. Middle rectal body of S2-4 spinal nerves
D. Uterine
E. * Iliolumbar 129. The rectum receives its blood supply primarily by the
branches of the:
123. Which of the following statements about the sacral plexus is A. Superior mesenteric artery
true? B. * Inferior mesenteric artery
A. It takes form on the anterior wall of the pelvis C. Inferior rectal arteries
B. Its major part lies on the obturator internus muscle D. Branches of the external iliac artery
C. * It gives off the pudendal nerve E. Femoral artery
D. Most of its branches pass above the piriformis to appear in the
buttock 130. Which one of the following statements is incorrect about
E. It supplies the medial cutaneous area below the knee innervation of the rectum?
A. * Its motor fibers include sympathetics
124. Which of these nerves is formed by posterior divisions of B. Its motor fibers are conveyed in the middle rectal plexus
the sacral plexus? C. Its afferent supply of nerves belongs to the parasympathetic
A. * Common peroneal system
B. Anococcygeal D. Pelvic splanchnic nerves are involved in its innervation
C. Pudendal E. The superior rectal plexus may supply rectal blood vessels
D. Nerve to the quadratus femoris
E. Nerve to the obturator internus and superior gemellus 131. All of these statements are true about the urinary bladder
except that:
125. Which of these nerves is formed by anterior divisions of the A. Its base is the superior posterior surface
sacral plexus? B. The median umbilical ligament is attached to its apex
A. Nerve to the piriformis C. Its neck leads into the urethra
B. Lateral part of the posterior femoral cutaneous nerve D. * When distended, it becomes lower in position
C. Superior gluteal E. Its body shows superior and inferolateral surfaces
D. * Tibial
E. Inferior gluteal 132. Which of these structures is not a component of the bladder?
A. Vesical trigone
Clinical Anatomy: An Illustrated Review with Questions and Explanations by Richard Snell 4th Ed.

B. Detrusor muscle
C. Pubovesical muscle 140. Which of these is characteristic of the vagina?
D. Ureteric ostia A. Its anterior fornix is deeper than the posterior fornix
E. * Vesical sphincter B. It terminates at the urogenital diaphragm
C. * It fuses around the cervix of the uterus
133. Correct relationships of the female ureters include which of D. It allows little distention
the following? E. Normally it is flattened laterally
A. The ureters pass lateral to the vagina
B. The ureters are not in close proximity to the vesical nerve 141. The greater part of the pelvic diaphragm is formed from the:
plexus A. Obturator internus muscle
C. * At the base of the broad ligament the uterine artery crosses B. Pelvic fascia
above and in front of the ureters C. Perineal membrane
D. The ureters enter the anterior aspect of the bladder D. * Levator ani
E. At the pelvic brim the ovarian vessels cross medial to the E. Coccygeus
ureters
142. To name visceral branches of the internal iliac artery:
134. The male urethra traverses all of the following structures A. * Middle rectal
except the: B. Obturator
A. Prostate gland C. Iliolumbar
B. * Ejaculatory duct D. Lateral sacral
C. Urogenital diaphragm E. Internal pudendal
D. Sphincter urethra
E. Internal urethral orifice 143. Branches of the internal iliac artery to the pelvic wall and
lower limb include which of the following?
135. Which of these statements correctly describes the seminal A. Inferior vesical
vesicles? B. Superior vesical
A. They store sperm C. Umbilical
B. * They secrete a seminal fluid component D. Uterine
C. They lie medial to the ductus deferens E. * Superior gluteal
D. They form the ampulla of the ductus deferens
E. They empty directly, and alone, into the prostate 144. Which of the following statements correctly describes the
urogenital region of the perineum?
136. The ovarian artery is a branch of the:
A. Internal iliac A. It contains the rectum
B. External iliac B. It contains the ischiorectal fossa
C. Inferior epigastric C. It includes the coccygeal triangle
D. * Abdominal aorta D. * It is pierced by the urethra
E. Uterine artery E. Its floor is the obturator internus muscle

137. Which one of these structures is not a part of the female 145. All of the following are correct statements about the pelvic
internal genital system? splanchnic nerves except which of these items?
A. Ovary A. They are parasympathetic nerves
B. * Perineal body B. They arise from cord levels S2, S3, S4
C. Mesovarium C. * They are the sole innervation of prostatic musculature
D. Infundibulum D. They contribute to the formation of the pelvic plexus
E. Vagina E. They are the sole motor fibers to the rectum

138. Which of these nerves is not included in the uterovaginal 146. Which of the following arteries is the chief blood supply of
nerve plexus? the perineum?
A. Sympathetics A. Obturator
B. Parasympathetics B. Superior gluteal
C. * Somatic C. Iliolumbar
D. Afferent D. External iliac
E. Vasomotor E. * Internal pudendal

139. Lymphatic drainage of the uterus includes all the following 147. The major innervation of the perineum is which of these
except the: nerves?
A. External iliac nodes A. Sciatic
B. * Inferior mesenteric nodes B. * Pudendal
C. Internal iliac nodes C. Femoral
D. Lumbar nodes D. Obturator
E. Superficial inguinal nodes E. Inferior gluteal
Clinical Anatomy: An Illustrated Review with Questions and Explanations by Richard Snell 4th Ed.

D. The ureter enters the superior aspect of the bladder


148. The morphological homologue in the female of the penis in E. * In the female the ureter crosses behind and below the uterine
the male is the: artery
A. * Clitoris
B. Labia majora 156. Which of the following structures does not attach into the
C. Labia minora perineal body?
D. Vestibular bulbs A. Perineal membrane
E. Vulva B. * Ischiocavernosus.
C. Sphincter ani externus
149. Most perineal structures send their lymphatics to the: D. Transversus perinei superficialis
A. Internal iliac nodes E. Bulbospongiosus
B. Nodes of the lumbar chain
C. * Superficial inguinal nodes 157. Which of these statements correctly describes the
D. External iliac nodes bulbourethral glands?
E. Inferior mesenteric nodes A. They are found only in the female
B. They open into the membranous urethra
150. Which one of these statements about the ischiorectal fossa is C. * They are located in the deep perineal pouch
incorrect? D. They are located close to the sides of the anus
A. They are spaces on each side of the anal canal E. Their ducts open into the scrotum
B. They contain the pudendal nerve and internal pudendal vessel
C. They contain connective tissue and fat 158. Territories supplied by the pudendal nerve include all the
D. * Their inferior walls are formed by the levator ani muscle following except the:
E. They are bordered laterally by the obturator internus muscle A. Anal triangle
B. * Pelvic surface of the levator ani
151. The inferior pelvic aperture is bordered by all the following C. Urogenital triangle
except the: D. Labium majus
A. Inferior margin of the pubic symphysis E. Scrotum
B. Rami of the pubis and ischium
C. Coccyx 159. Which of the following is a branch of the internal pudendal
D. Sacrotuberous ligament artery?
E. * Iliac crest A. Superior gluteal
152. The urogenital diaphragm includes which of these B. Uterine
structures? C. Inferior vesical
A. A continuous sheet of smooth muscle D. Umbilical
B. The anal opening
C. * Superior and inferior fascial E. * Inferior rectal
D. The coccygeus
E. The obturator internus muscle 160. Correct statements regarding the middle rectal vein include
which of these?
153. The deep perineal space is described correctly by which of A. It begins in the anal columns
the following statements? B. It becomes the inferior mesenteric vein
A. * It is filled completely by the deep perineal muscles C. It has no valves
B. In the male it surrounds the scrotum and penis D. * It is a more important vessel than the middle rectal artery
C. It is limited by the superficial fascia E. It drains the rectum below the internal sphincter
D. Its fascia is a continuation of abdominal fascia
E. It communicates directly with the ischiorectal fossa 161. The superior gluteal artery is a branch of the:
A. Obturator
154. The superficial perineal space includes which one of these B. * Internal iliac
structures? C. Internal pudendal
A. Deep transverse perineal muscle D. Iliolumbar
B. Puborectalis E. Middle rectal
C. Sphincter urethrae
D. * Superficial perineal fascia 162. Branches of the sacral plexus include which of these nerves?
E. Perineal body A. Obturator
B. Femoral
155. Which of these statements correctly describes the course of C. * Sciatic
the ureter? D. Iliohypogastric
A. The right ureter is related to the base of the sigmoid colon E. Genitofemoral
B. After crossing the pelvic brim, the ureter lies lateral to the
internal iliac artery 163. Which of the following items correctly describes the pelvic
C. In the male, as it nears the bladder, it passes above the ductus splanchnic nerves?
deferens A. They send motor branches to the coccygeus
Clinical Anatomy: An Illustrated Review with Questions and Explanations by Richard Snell 4th Ed.

B. They contract the sphincter ani internus E. Innervation is not necessary to the function of this organ
C. They are sensory to the perineum
D. They cause contraction of the arteries of the penis 171. Which of the following structures is most important for
E. * They arise from spinal levels S2, S3, S4 support of the female pelvic viscera?
A. The supporting fasciae
164. Which of these structures is attached to the posterior aspect B. The perivascular stalk as a suspensory structure
of the broad ligament? C. The uterosacral ligament
A. * Ligament of the ovary D. * The levator ani muscle
B. Uterine tube E. The round ligament of the uterus
C. Round ligament of the uterus
D. Uterine artery 172. Functions of the pelvic splanchnic nerves include which of
E. Ureter these?
A. To contract arteries of the erectile tissue
165. Which of the following nerves passes through the greater B. To carry motor fibers to striated muscle
sciatic foramen and largely innervates the perineum? C. To supply muscle of the prostate
A. * Pudendal D. To carry sensory fibers from the body of the uterus
B. Superior gluteal E. * To control muscular walls of the bladder
C. Inferior gluteal
D. Sciatic 173. Pelvic lymph nodes include all the following except the:
E. Posterior cutaneous nerve of the thigh A. Common iliac
B. Internal iliac
166. All of the following statements correctly describe the ovary C. Lateral sacral
except: D. * Superior hypogastric
A. It is covered with cuboidal epithelium E. Median sacral
B. Its anterior border is attached to the broad ligament by the
mesovarium 174. Which of these statements describes the extent of the
C. The suspensory ligament of the ovary suspends tts tubal pole perineum?
D. * It has a smooth surface A. From the symphysis pubis to the central perineal tendon
E. The ligament of the ovary attaches it to the lateral margin of B. * From the symphysis pubis to the tip of the coccyx
the uterus C. From the central perineal tendon to the tip of the coccyx
D. From the ischial tuberosities to the symphysis pubis
167. The abdominal orifice of the uterine tube is located at its:
A. Ampulla E. From the symphysis pubis to the anus
B. Isthmus
C. Uterine part 175. The anterior half of the inferior pelvic aperture is closed by
D. Fundus which of these structures?
E. * Infundibulum A. The levator ani muscle
B. The coccygeus muscle
168. The part of the uterus that rises above the uterine tubes is C. * The urogenital diaphragm
the: D. The arcuate pubic ligament
A. External os E. The urethral sphincter muscle
B. Body
C. * Fundus 176. All of the following structures are found in the area of the
D. Cervix urogenital triangle except the:
E. Fornix A. * External anal sphincter
B. Sphincter urethrae muscle
169. Correct relations of the female ureters include which of the C. Arcuate pubic ligament
following? D. Deep transversus perinei muscle
A. They are equidistant from each side of the cervix E. Perineal membrane
B. * They cross the lateral fornix of the vagina
C. They cross above the broad ligament 177. The wedge-shaped mass of fibrous tissue located at the
D. They enter the bladder behind the vagina center of the perineum is the
E. They cross above the uterine artery A. Urorectal septum
B. Perineal membrane
170. These statements regarding innervation of the uterus are C. Ischiorectal tendon
correct except for which one of the following? D. Sacrotuberous ligament
A. * It includes many parasympathetic efferents E. * Perineal body
B. The uterovaginal plexus holds many sympathetic efferent fiber
C. Afferents from the body of the uterus travel with the 178. In the male, the superficial perineal space contains all
sympathetic system except which one of these structures?
D. Pain fibers from the uterus enter the spinal cord through the A. The root of the penis
last two thoracic nerves B. Branches of internal pudendal vessels
Clinical Anatomy: An Illustrated Review with Questions and Explanations by Richard Snell 4th Ed.

C. The proximal part of the spongy urethra A. Is composed of erectile tissue


D. * The membranous urethra B. Is composed of two crura
E. The pudendal nerve C. Has two corpora cavernosa
D. * Has a corpus spongiosum
179. In the female, the deep perineal space contains all except E. Is suspended by a suspensory ligament
which one of the following structures?
A. * The greater vestibular glands 187. Which of the following statements correctly compares the
B. Part of the urethra female pelvis with the male pelvis?
C. The inferior part of the vagina A. The hip bones are closer together than in the male
D. The deep transverse perineal muscles B. The sacrum is narrower than in the male
E. The sphincter urethrae C. * There are fewer prominent bony markings than in the male
D. The ischial tuberosities are closer together than in the male
180. The ischiorectal fossae are described correctly by which of E. The subpubic angle of the pubic arch is less than in the male
these statements?
A. There is no communication between the two sides 188. The diagonal conjugate diameter of the pelvis is a
B. * They are located on each side of the rectum measurement of the superior pelvic aperture between which of
C. They are filled with watery fluid the following points?
D. Anteriorly they continue inferior to the urogenital diaphragm A. * The midpoint of the inferior border of the symphysis pubis
E. They have no relation to rectal function to the midpoint of the sacral promontory
B. The midpoint of the superior border of the symphysis pubis to
181. Which of the following nerves does not innervate the the midpoint of the sacral promontory
scrotum? C. Transversely from the linea terminalis on one side to this line
A. The ilioinguinal on the opposite side
B. Medial branches of the perineal D. From one iliopubic eminence to the opposite sacroiliac joint
C. Branches of the pudendal E. From one ischial spine to the opposite ischial spine
D. * Inferior gluteal
E. Posterior femoral cutaneous 189. A pelvis in which the anteroposterior (AP) diameter of the
superior pelvic aperture is short and the transverse diameter is
182. Most of the innervation of the perineum is supplied by long is called:
which of these nerves? A. Gynecoid
A. Obturator B. Anthropoid
B. * Pudendal C. * Platypelloid
C. Femoral D. Android
D. Sciatic E. Piriform
E. Sympathetics
190. Which of these muscles crosses the pelvic brim?
183. The deep perineal space of the male contains which of these A. Piriformis
structures? B. Obturator interims
A. * Bulbourethral glands C. Levator ani
B. Bulbospongiosus muscle D. Coccygeus
C. Ischiocavernosus muscle E. * None
D. The spongy urethra
E. The root of the penis 191. Which of these muscles is a part of the pelvic diaphragm?
A. Obturator intemus
184. The blood supply of the scrotum includes which of these B. Piriformis
arteries? C. * Puborectalis
A. Inferior vesical D. Quadratus femoris
B. Umbilical E. Deep transverse perineal muscle
C. * External pudendal
D. Inferior gluteal 192. Which of these nerves does not arise from the sacral plexus?
E. Superior rectal A. Sciatic
B. * Obturator
185. Which of the following structures is not located in the C. Pudendal
female superficial perineal space? D. Superior gluteal
A. Superficial transverse perineal muscle E. Inferior gluteal
B. Ischiocavernosus muscle
C. Bulbospongiosus muscle 193. All of the following arteries enter the pelvis minor except
D. * Greater vestibular glands the:
E. Dorsal nerve of the clitoris A. Internal iliac (paired)
B. Median sacral
186. The clitoris of the female is similar to the penis of the male C. * Femoral
except that it: D. Superior rectal
Clinical Anatomy: An Illustrated Review with Questions and Explanations by Richard Snell 4th Ed.

E. Ovarian (paired) D. Prostate


E. Bulbourethral gland
194. The superior rectal artery is a branch of which of these
arteries? 202. Correct statements about the seminal vesicles include which
A. * Inferior mesenteric of the following?
B. Umbilical A. * Their ducts contribute to formation of the ejaculatory duct
C. Inferior vesical B. Their secretion probably is not valuable
D. Internal, pudendal C. They are situated posterior to the rectum
E. Middle rectal D. They store sperm
E. They lie in front of the ureters
195. In the female, which of these statements about the route of
the pelvic part of the ureter is correct? 203. Descriptions of the position of the uterus include all but
A. It descends on the anterior wall of the pelvis which one of these?
B. It forms the anterior boundary of the ovarian fossa A. Anteversion
C. It passes lateral to the origin of the uterine artery B. Anteflexion
D. * At the level of the ischial spine, it is crossed superiorly by C. * Vertical proflexion
the uterine artery D. Retroversion
E. It enters the anterior angle of the bladder E. Retroflexion

196. The base of the urinary bladder is its: 204. Pain from the ovary is usually referred to which of the
A. * Posterior surface following regions?
B. Anterior end A. * Inguinal and pubic regions
C. Inferolateral surface B. Perineum, posterior thigh, and leg
D. Neck C. Shoulder
E. Superior surface D. Back
E. None of the these
197. Arteries of the urinary bladder include all except which one
of the following? 205. Pain associated with external hemorrhoids is mediated by:
A. Superior vesical A. The hypogastric nerves to the lumbar splanchnic nerves
B. Inferior vesical B. The pelvic splanchnic nerves
C. * External iliac C. * The pudendal nerve
D. Inferior gluteal D. The sacral sympathetic chain
E. Obturator E. None of the above

198. The ejaculatory ducts open into which of the following 206. Laceration of the male urethra just inferior to the urogenital
structures? diaphragm would likely result in extravasation of urine into all
A. External urethral orifice the following regions except?
B. Urethral crest A. The abdominal wall between the deep layer of superficial
C. Ureter fascia and deep fascia
D. * Prostatic urethra B. * The anal triangle between the superficial (ischioahal) fat and
E. Urinary bladder the deep fascia
C. The penis between the superficial fascia and the deep (Buck's)
199. All of these are true statements about the female urethra fascia
except which one of the following? D. The scrotum deep to the dartos layer
A. It corresponds to the prostatic and membranous parts of the E. The urogenital triangle between the deep layer of superficial
male urethra (Colles') fascia and deep fascia
B. * It is longer than the male urethra 207. Cancer from the testis would most likely metastasize first to
C. It passes through the pelvic diaphragm the:
D. Inferiorly it is associated intimately with the vagina A. * Lumbar nodes
E. It passes through the urogenital diaphragm B. Superficial inguinal nodes
C. Deep inguinal nodes
200. Which of these structures is a male accessory genital gland? D. Internal iliac nodes
A. Testes E. External iliac nodes
B. Deferent ducts
C. Seminal vesicles 208. Hydrocele is a fluid accumulation within which of the
D. Ejaculatory ducts following?
E. * Prostate A. Round ligament
B. Gubernaculum testis
201. The ductus deferens ends in which of the following? C. Scrotal ligament
A. Testes D. * Tunica vaginalis testis
B. Seminal vesicles E. Vas deferens
C. * Ejaculatory duct
Clinical Anatomy: An Illustrated Review with Questions and Explanations by Richard Snell 4th Ed.

209. In the male, obstruction of the ureter by ureteric stones


occurs most often at which of the following portions of this 217. The middle rectal artery originates from the:
structure? A. Prasacral artery
A. Nearest the kidney B. Common iliac artery
B. On the side wall of the pelvis C. * Internal iliac artery
C. Just superior to the ischial spine D. External iliac artery
D. Just lateral to the ductus deferens E. Inferior mesenteric artery.
E. * Where it crosses the external iliac artery and the brim of the
pelvis 218. The obturator internus muscle receives its nerve supply
from the:
210. The nerve most likely to be injured during removal of A. Obturator nerve
cancerous lymph nodes from the side wall of the pelvis is the: B. Pudendal nerve
A. Femoral C. Pudendal nerve and the perineal branch of S4
B. Lumbosacral trunk D. Hypogastric plexus
C. Sciatic E. * None of the these.
D. Pudendal
E. * Obturator 219. The internal anal sphincter receives its nerve supply from
the:
211. Infection may reach the ischiorectal fossa from all except A. * Hypogastric plexuses
which one of the following? B. Vagus nerve
A. Inflammation of the anal sinuses C. Obturator nerve
B. Downward extension of a perirectal abscess D. Pudendal nerve
C. Following a tear in the anal mucous membrane E. Pudendal nerve and the perineal branch of S4
D. A penetrating wound in the anal region
E. * A ruptured urethra 220. The sphincter urethrae receives its innervation from the:
A. Vagus nerve
212. A carcinoma of the cervix of the uterus is likely to spread B. Obturator nerve
via the lymphatics into the: C. * Pudendal nerve
A. External iliac nodes D. Inferior rectal nerve
B. Internal iliac nod E. Hypogastric plexuses.
C. Superficial inguinal nodes
D. * Internal and external iliac nodes 221. The levator ani muscle receives its innervation from the:
E. Presacral lymph nodes. A. Pudendal nerve
B. Hypogastric plexuses
213. A carcinoma of the prostate is likely to spread via the C. * Pudendal nerve and the perineal branch of S4
lymphatics into the: D. Obturator nerve
A. Internal and external iliac nodes E. Inferior rectal nerve.
B. * Internal iliac nodes
C. Para-aortic nodes 222. The superior rectal vein drains into the:
D. Superficial inguinal nodes A. Inferior vena cava
E. Inferior mesenteric nodes. B. Internal iliac vein
C. External iliac vein
214. The superior rectal artery originates from the: D. * Inferior mesenteric vein
A. Internal pudendal artery E. None of these.
B. External iliac artery
C. Internal iliac artery 223. The right ovarian vein drains into the:
D. * Inferior mesenteric artery A. Right internal iliac vein
E. None of these. B. * Inferior vena cava
C. Inferior mesenteric vein
215. The left ovarian artery originates from the: D. Right external iliac vein
A. External iliac artery E. Right renal vein.
B. Internal iliac artery
C. Left renal artery 224. A patient was examined with a left-sided varicocele; the left
D. Left common iliac artery testicular vein drains into the:
E. * Abdominal part of the aorta. A. * Left renal vein
B. Left internal iliac vein
216. The inferior rectal artery originates from the: C. Left external iliac vein
A. Superior mesenteric artery D. Inferior vena cava
B. Internal iliac artery E. Left inferior suprarenal vein.
C. * Internal pudendal artery
D. External iliac artery 225. The prostatic venous plexus drains into the:
E. Presacral artery. A. Inferior vena cava
Clinical Anatomy: An Illustrated Review with Questions and Explanations by Richard Snell 4th Ed.

B. * Internal iliac vein C. Anterior to the prostatic urethra


C. External iliac vein D. Lateral to the lower part of the prostatic urethra
D. Internal and external iliac veins E. * Superior to the ejaculatory ducts and posterior to the upper
E. Testicular veins. part of the prostatic urethra.

226. In most women, the anatomic position of the uterus when 234. The promontory of the sacrum is formed by the:
the bladder is empty is A. Anterior surface of the second sacral vertebra
A. Retroverted B. Lower border of the anterior surface of the fifth sacral
B. Anteverted vertebra
C. Anteflexed C. Lateral masses of the sacrum
D. * Anteverted and anteflexed D. * Anterior and upper border of the first sacral vertebra
E. Retroflexed. E. Lateral edge of the sacrum.

227. The uterus receives its blood supply from the: 235. The pelvic outlet is bounded posteriorly by the coccyx,
A. Superior vesical artery laterally by the sacrotuberous ligamentous and the _______ , and
B. Middle rectal artery anteriorly by the pubic arch:
C. Ovarian artery A. Ischial spines
D. Uterine artery B. Piriformis muscle
E. * Uterine and ovarian arteries. C. * Ischial tuberosities
D. Perineal membrane
228. In a woman with ovarian cancer, it is judicious to examine E. Obturator foramen.
the:
A. Peritoneal cavity for evidence of excessive fluid (ascites) 236. The pelvic diaphragm is formed by the ______ and
B. Superficial inguinal lymph nodes coccygeus muscles and their covering fasciae:
C. Para-aortic nodes at the level of the first lumbar vertebra A. Piriformis
D. External iliac lymph nodes B. * Levator ani
E. * Para-aortic nodes and evidence of excessive peritoneal fluid C. Deep transverse perineal muscles
(ascites). D. Perineal membrane
E. Sphincter urethrae.
229. The narrowest part of the male urethra is the:
A. Membranous part 237. The urogenital diaphragm is attached laterally to the:
B. Prostatic part A. Tip of the coccyx
C. Penile part B. Ischial spine
D. * External meatus on the glans penis C. * Inferior ramus of the pubis and the ischial ramus
E. None of the above. D. Ischial tuberosities
E. Obturator internus fascia.
230. The posterior wall of the lower third of the vagina is
supported by the: 238. During defecation, the levator ani muscles:
A. Coccyx A. Are completely inactive
B. * Perineal body B. Do not support the uterus and vagina
C. Anal canal
D. Anococcygeal body C. * Relax (puborectalis portion) with the anal sphincter
E. None of the above. D. Do not support the sigmoid colon
E. Do not support the bladder.
231. The rectouterine pouch (pouch of Douglas) can be most 239. The anal columns:
efficiently entered by a surgical incision through the: A. * Contain tributaries of the superior rectal vein
A. * Posterior fornix of the vagina B. Are connected at their ends by spiral valves
B. Anterior fornix of the vagina C. Are located in the lower half of the anal canal
C. Anterior rectal wall D. Are transverse folds of mucous membrane
D. Lateral fornix of the vagina E. Contain branches of the pudendal nerve.
E. Posterior wall of the cavity of the uterine body.
240. The mucous membrane lining the upper half of the anal
232. Cancer of the prostate can metastasize to the skull via the: canal is:
A. Pampiniform plexus A. Lined with stratified squamous epithelium
B. External iliac veins B. Drained by the inferior rectal vein
C. * Vertebral venous plexus C. Drained into the superficial inguinal lymph nodes
D. Portal vein D. Sensitive to touch and to pain
E. Inferior vena cava. E. * Sensitive to stretch.

233. The middle (median) lobe of the prostate lies: 241. The rectouterine pouch (pouch of Douglas):
A. Inferior to the ejaculatory ducts A. Is formed by parietal pelvic fascia
B. Superior to the ejaculatory ducts B. Commonly contains coils of jejunum
Clinical Anatomy: An Illustrated Review with Questions and Explanations by Richard Snell 4th Ed.

C. Lies anterior to the vagina 248. The following part or branch of the brachial plexus receives
D. * Lies behind the posterior fornix of the vagina and the body contributions from the C8 spinal nerve:
of the uterus A. Lateral cord
E. Is not the most dependent part of the female peritoneal cavity B. Lateral pectoral nerve
when the woman is in the standing position. C. * Posterior cord
D. Nerve to the subclavius muscle
242. The following structures are closely relates to the E. Suprascapular nerve.
rectouterine pouch (pouch of Douglas) except which?
A. Anteriorly is situated the posterior surface of the upper part of 249. The following part or branch of the brachial plexus has a
the vagina terminal branch that supplies the skin on the medial side of the
B. Posteriorly is situated the upper part of the rectum arm:
C. * The trigone of the bladder is directly related to its anterior A. Musculocutaneous nerve
wall B. Lateral cord
D. Anteriorly is situated the posterior surface of the body of the C. Thoracodorsal nerve
uterus D. * Medial cord
E. Laterally is situated the sacrocervical ligaments passing E. Ulnar nerve.
forward to the cervix.
250. The following part of the brachial plexus is formed from the
243. The following statements regarding the ovary are correct anterior divisions of two trunks:
except which? A. * Lateral cord
A. * It is attaches to the lateral pelvic wall by the round ligament B. Posterior cord
of the ovary C. Median nerve
B. It normally is not related to the posterior fornix of the vagina D. Medial cord
C. The right ovarian vein drains into the inferior vena cava. E. None of the above.
D. It is attached to the posterior layer of the broad ligament
E. It ovulates an ovum into the peritoneal cavity 251. The following part of the brachial plexus has branches that
supply the extensor muscles of the arm:
244. In males, traumatic injury to the perineum may rupture the A. Lateral cord
bulb of the penis or the penile urethra. The resulting leakage of B. * Posterior cord
blood or urine may be found in all of the following areas except C. Median nerve
which? D. Medial cord
A. The anterior abdominal wall E. Lateral and medial cords.
B. * The ischiorectal fossa
C. The scrotum 252. During its course in the upper limb, the axillary nerve lies:
D. The penis A. In front of the lateral epicondyle of the humerus
E. The superficial perineal pouch. B. Against the spiral groove of the humerus
C. Medial to the brachial artery in the cubital fossa
245. Using your knowledge of anatomy, explain the relationship D. * Against the surgical neck of the humerus
of the uterine tube to the peritoneal cavity; E. In front of the medial epicondyle of the humerus.
A. The tube is retroperitoneal
B. The tube lies within the peritoneal cavity 253. During its course in the upper limb, the ulnar nerve lies:
C. The tube lies within the broad ligament near its base A. In front of the lateral epicondyle of the humerus
D. * The tube lies in the free margin of the broad ligament B. Against the spiral groove of the humerus
E. The tube lies within the parametrium. C. Medial to the brachial artery in the cubital fossa
D. Against the surgical neck of the humerus
246. The tender, doughlike mass felt through the posterior wall E. * Behind the medial epicondyle of the humerus.
of the vagina resulted from what?
A. A retroverted uterus 254. During its course in the upper limb, the median nerve lies:
B. * Blood in the pouch of Douglas A. Anterior to the flexor retinaculum of the wrist
C. A full bladder B. In front of the lateral epicondyle of the humerus
D. Blood in the uterovesical pouch C. * Medial to the brachial artery in the cubital fossa
E. A prolapsed ovary. D. Within the quadrangular muscle space
E. Against the surgical neck of the humerus.
247. What is the venous drainage of the mucous membrane of the
anal canal? 255. During its course in the upper limb, the radial nerve lies:
A. Middle rectal veins only A. * Against the spiral groove of the humerus
B. Inferior rectal veins only B. In front of the lateral epicondyle of the humerus
C. * Superior and inferior rectal veins C. Medial to the brachial artery in the cubital fossa
D. Middle and inferior rectal veins D. Within the quadrangular muscle space
E. Internal pudendal veins. E. Against the surgical neck of the humerus.
Clinical Anatomy: An Illustrated Review with Questions and Explanations by Richard Snell 4th Ed.

256. The extensor carpi radialis brevis muscle is innervated by 264. To test for trapezius muscle paralysis, you would ask the
the patient to:
A. Radial nerve A. Flex the arm fully
B. Ulnar nerve B. Adduct the arm against resistance
C. Superficial radial nerve C. Push against the wall with both hands
D. * Deep branch of the radial nerve D. * Shrug the shoulder
E. Deep branch of the ulnar nerve. E. Abduct the arm fully.

257. The dorsal interossei muscles are innervated by the: 265. Cutting the dorsal scapular nerve would most likely result in
A. Deep branch of the radial nerve paralysis of the:
B. * Deep branch of the ulnar nerve A. Supraspinatus muscle
C. Musculocutaneous nerve B. Deltoid muscle
D. Median nerve C. * Rhomboid major muscle
E. Recurrent branch of the median nerve. D. Trapezius muscle
E. Infraspinatus muscle.
258. The extensor indicis muscle is innervated by the:
A. Radial nerve 266. After injury to a nerve at the wrist, the thumb is laterally
B. Ulnar nerve rotated and adducted. The hand looks flattened and apelike. The
C. Median nerve nerve that has been damaged is the:
D. * Deep branch of the radial nerve A. Anterior interosseus nerve
E. Deep branch of the ulnar nerve. B. Ulnar nerve
C. Deep branch of the radial nerve
259. The extensor carpi ulnaris muscle is innervated by the: D. * Median nerve
A. Radial nerve E. Superficial branch of the radial nerve.
B. * Deep branch of the radial nerve
C. Ulnar nerve 267. The following statements concerning the blood vessels of
D. Superficial branch of the ulnar nerve the upper limb are correct except which?
E. Median nerve. A. The pulsation of the radial artery are felt anterior to the distal
third of the radius between the tendons of the brachioradialis and
260. The extensor carpi radialis longus muscle is innervated by the flexor carpi radialis
the: B. * The cephalic vein arises on the palm of the hand
A. Deep branch of the ulnar nerve C. The axillary vein is formed by the union of the venae
B. Ulnar nerve comitantes of the brachial artery and the basilic vein
C. * Radial nerve D. The axillary sheath surrounds the axillary vessels and the
D. Median nerve brachial plexus
E. Anterior interosseus nerve. E. The cephalic vein drains into the axillary vein.

261. A shoulder separation that involves the lateral end of the 268. At the wrist, the flexor retinaculum is attached to the
clavicle sliding onto the superior aspect of the acromion would following bones except which?
most likely result from damage to the: A. The hook of the hamate
A. Costoclavicular ligament B. The ridge on the trapezium
B. Sternoclavicular ligament C. The pisiform bone
C. * Coracoclavicular ligament D. The tubercle of the scaphoid
D. Glenohumeral ligament E. * The triquetral bone.
E. Coracoacromial ligament.
269. The following statements concerning the shoulder joint are
262. The muscle that will compensate in part for the paralysis of correct except which?
the supinator muscle is the:
A. Extensor carpi ulnaris muscle A. The inferior part of the capsule is the weakest
B. Brachialis muscle B. * The subacromial bursa communicates with the joint cavity
C. Triceps brachii muscle C. The strength of the joint depends largely on the tone of the
D. * Biceps brachii muscle surrounding muscles
E. Anconeus muscle. D. Dislocation of the shoulder joint is common
E. The axillary artery lies in front of the shoulder joint.
263. The synovial sheath of the flexor pollicis longus muscle
forms the: 270. The following statements concerning the lymphatic drainage
A. Thenar space of the upper limb are correct except which?
B. * Radial bursa of the wrist A. Lymph from an infected nail bed of the little finger drains into
C. Midpalmar space the supratrochlear lymph node
D. Ulnar bursa of the wrist B. Lymph from infected cut of the index finger drains into the
E. Digital synovial sheath for the index finger. infraclavicular nodes
Clinical Anatomy: An Illustrated Review with Questions and Explanations by Richard Snell 4th Ed.

C. * Lymph from an infected graze over the medial side of the


elbow joint drains into the anterior (pectoral) group of axillary 277. The following statements concerning the lateral cord of the
nodes brachial plexus are true except which?
D. Lymph from the upper lateral quadrant of the breast drains A. It contains sympathetic nerve fibers
into the anterior (pectoral) group of axillary nodes B. It has a branch that supplies the pectoralis major muscle
E. Lymph from a boil on the back over the inferior angle of the C. It has a branch that supplies the skin on the lateral side of the
scapula drains into the posterior (subscapular) nodes. forearm
D. * It has a branch that supplies the skin on the lateral side of
271. The following structures pass superficial to the flexor the upper arm
retinaculum at the wrist except which? E. It lies lateral to the second part of the axillary artery.
A. Palmar cutaneous branch of the median nerve
B. Ulnar nerve 278. An examination of a patient with carpal tunnel syndrome
C. * Flexor pollicis longus tendon may reveal all the following symptoms and signs except which?
D. Ulnar artery A. Atrophy of the muscles of the thenar eminence
E. Palmar cutaneous branch of the ulnar nerve. B. Weakness in opposition of the thumb
C. * Loss of skin sensation on the medial part of the palm
272. Collateral circulation around the shoulder joint would D. Loss of skin sensation on the anterior surface of the index
involve the following except which? finger
A. The subscapular artery E. Normal skin sensation on the anterior surface of the little
B. The superficial cervical artery finger.
C. The suprascapular artery
D. The anterior circumflex humeral artery 279. The following movements are expected to be normal after
E. * The lateral thoracic artery. complete section of the medial cord of the brachial plexus except
which?
273. The proximal row of carpal bones includes all the following A. Extension of the wrist
carpal bones except which? B. Flexion of the elbow
A. The pisiform C. Abduction of the shoulder joint
B. * The capitate D. * Metacarpophalangeal flexion and interphalangeal extension
C. The lunate of the medial four fingers
D. The triquetral E. Metacarpophalangeal flexion and interphalangeal extension of
E. The scaphoid. the thumb.

274. Regarding a “winged scapula”, the following facts are 280. The following statements regarding the sympathetic
correct except which? innervation of the upper limb are correct except which?
A. * The spinal part of the accessory nerve is damaged A. There are preganglionic nerve fibers originating in spinal cord
B. The inferior angle of the scapula projects backward segments T2 to 8
C. The serratus anterior muscle may be wasted B. It causes vasoconstriction of the arteries and veins of the skin
D. The long thoracic nerve is damaged C. There are preganglionic nerve fibers synapsing in the middle
E. The scapula can no longer be pulled anteriorly around the cervical, the inferior cervical, and the first thoracic ganglia
chest wall (as in thrusting the upper limb anteriorly when D. Many of the postganglionic fibers are distributed within the
reaching). branches of the brachial plexus
E. * The sympathetic nerves do not innervate the sweat glands.
275. The following statements concerning the rotator cuff are
correct except which: 281. During an automobile accident, a patient fractured the neck
A. It adds stability to the shoulder joint of her right radius and damaged a closely related nerve. At
B. It is formed by the tendons of the short muscles around the physical examination, the patient exhibited the following except
shoulder joint which?
C. The muscle tendons are fused to the capsule of the shoulder A. Weakness in extending the terminal phalanx of the thumb
joint B. * A loss of skin sensation on the lateral part of the dorsum of
D. Degeneration or tearing of the cuff will cause severe pain in the hand
the shoulder region C. An inability to extend the metacarpophalangeal joint of the
index finger
E. * All the muscle tendons associated with the cuff are D. A normal ability to adduct the thumb at the carpometacarpal
innervated by the suprascapular nerve. joint
E. Normal skin sensation down the medial border of the hand.
276. The quadrangular space in the region of the shoulder
transmits the following structures except which? 282. Diminished sweating and increased warmth and
A. The axillary nerve vasodilatation of the skin vessels over the hypothenar eminence
B. The posterior circumflex humeral artery as well as the ring and the little fingers could result from the
C. The lymphatic vessels following except which?
D. * The radial nerve A. A lesion of the posterior cord of the brachial plexus
E. The posterior circumflex humeral vein.
Clinical Anatomy: An Illustrated Review with Questions and Explanations by Richard Snell 4th Ed.

B. Ulnar nerve damage behind the medial epicondyle of the C. A. circumflexa scapulae
humerus D. A. thoracodorsalis
C. A lesion of the medial cord of the brachial plexus E. A. thoracica lateralis.
D. A lesion of the eighth cervical nerve
E. * Ulnar nerve damage over the front of the wrist. 291. The first part of axillary artery is situated within which of
the following triangles:
283. Which of the following muscles was likely to have been A. * Claviculopectoral
paralyzed by the dislocation of the shoulder joint some years B. Pectoral
ago? C. Infrapectoral
A. The rhomboid minor D. Clavicular
B. * The deltoid E. Infradeltoid.
C. The supraspinatus
D. The teres minor 292. The second part of axillary artery is situated within which of
E. The subscapularis. the following triangles:
A. * Pectoral
284. The suprascapular artery arises from: B. Claviculopectoral
A. Inferior thyroid artery C. Infrapectoral
B. Ascending artery of neck D. Clavicular
C. * Thyrocervical trunk E. Infradeltoid.
D. Infrascapular artery
E. Circumflex artery of scapula. 293. The third part of axillary artery is situated within which of
the following triangles:
285. The medial cutaneous nerve of arm arises from: A. Pectoral
A. Lateral cord of brachial plexus B. Claviculopectoral
B. * Medial cord of brachial plexus C. * Infrapectoral
C. Posterior cord of brachial plexus D. Clavicular
D. Musculocutaneous nerve E. Infradeltoid.
E. Medianus nerve.
294. Which of the following arteries arise from the axillary artery
286. The posterior cutaneous nerve of arm arises from: within the pectoral triangle?
A. Lateral cord of brachial plexus A. * A. thoracica lateralis
B. Medial cord of brachial plexus B. A. transversae colli
C. * Posterior cord of brachial plexus C. A. circumflexa scapulae
D. Ulnar nerve D. A. thoracodorsalis
E. Radial nerve. E. A. circumflexa humeri anterior.

287. The superior lateral cutaneous nerve of arm arises from: 295. Which of the following arteries arise from the axillary artery
A. * Lateral cord of brachial plexus within the subpectoral triangle?
B. Medial cord of brachial plexus A. A. transversae colli, a. circumflexa scapulae, a. thoracica
C. Posterior cord of brachial plexus lateralis
D. Musculocutaneous nerve B. * A. circumflexa humeri anterior and posterior, a.
E. Axillary nerve. subscapularis
C. A. circumflexa scapulae, a thoracica lateralis
288. The superior ulnar collateral artery pass with which of the D. A. thoracodorsalis, a. circumflexa humeri anterior
following artery: E. A. circumflexa humeri anterior
A. Medianus nerve
B. * Ulnar nerve 296. Which of the following cords form the brachial plexus?
C. Radial nerve A. * Medial, lateral, posterior
D. Musculocutaneous nerve B. Medial, lateral, anterior
E. Posterior cutaneous nerve of arm. C. Medial, lateral
D. Superior, medial, lateral
289. Which of the following anatomic structures pass through the E. Inferior, medial, lateral.
carpal canal:
A. Medianus nerve 297. Pyrogov’s space is situated between which of the following
B. Ulnar nerve muscular layers of forearm:
C. Radial nerve A. I – II
D. Interosseal artery of palm B. II – III
E. * Superficial and profundus tendon of fingers. C. * III – IV
D. IV – V
290. Subscapular artery arises from: E. None of these.
A. * A. axillares
B. A. transversae colli
Clinical Anatomy: An Illustrated Review with Questions and Explanations by Richard Snell 4th Ed.

298. Which of the following anatomic structures form floor of


anatomic snuffbox? 306. Specify the zone of innervation of n. peroneus superficialis:
A. Os hamatum A. anterior group of leg muscles and skin of lateral surface of
B. Os pisiforme leg;
C. * Os naviculare B. lateral group of leg muscles and skin of anterior surface of leg;
D. Os capitatum C. skin of posterior surface of leg until tip of malleolus lateralis;
E. Os lunatum. D. muscles and skin of posterior surface of leg;
E. * lateral group of leg muscles and skin of lateral surface of leg
299. Which of the following bones can be palpated on floor of and foot;
anatomic snuffbox?
A. * Os hamatum 307. Damage of n. peroneus communis leads to which of the
B. Os pisiforme following forms of foot?
C. Os naviculare A. griffin claw;
D. Os capitatum B. most abducted;
E. Os lunatum. C. most flexed;
D. most adducted;
300. Which of the following vessels can be pressed within E. * pes equinus;
anatomic snuffbox?
A. * a.radialis 308. Specify the zone of innervation of n. peroneus profundus:
B. a.ulnaris A. lateral group of leg muscles, skin of II interdigital interval on
C. a.digitales dorsales foot;
D. a.digitales dorsales communes B. posterior group of leg muscles, skin of plantar surface of foot;
E. none of these C. anterior group of leg muscles;
D. skin of I interdigital interval on foot;
301. Which of the following joints has name “pelvic key”? E. * anterior group of leg muscles, skin of I interdigital interval
A. art. lumbosacralis; on foot;
B. art. sacrococcygealis;
C. symphysis; 309. Which of the following muscles is guideline for determining
D. all are false; the position of vascular-nerve bundle in the axillary fossa in case
E. * art. sacroiliaca; of abducted and rotated outward limb?
A. m. deltoideus;
302. N. tibialis goes out from popliteal fossa between which of B. m. pectoralis major;
the following muscles? C. m. pectoralis minus;
A. m. peroneus longus, m. peroneus brevis; D. caput longum of m. biceps brachii;
B. m. peroneus brevis, m. popliteus; E. * m. coracobrachialis;
C. m. tibialis posterior, m. plantaris;
D. m. flexor digitorum longus, m. peroneus brevis; 310. Which of the following muscles are intimately adherent to
E. * m. popliteus, m. plantaris; the capsule of the elbow joint?
A. m. coracobrachialis, m. supinator;
303. In the interval between which of the following muscles B. m. anconeus, m. supinator;
femoral artery goes to the posterior surface of the knee? C. m. pronator teres, m. supinator;
A. m. rectus femoris, m. gracilis; D. m. flexor carpi radialis;
B. m. vastus lateralis, m. biceps femoris; E. * m. brachialis and m. supinator;
C. m. adductor longus, m. biceps femoris;
D. m.adductor magnus, m. adductor minus; 311. Indicate the branches of 1st part of axillary artery:
E. * m. vastus medialis, m.adductor magnus; A. a. subclavia;
B. a. circumflexa scapulae;
304. Which of the following arteries arise from popliteal artery C. a. thoracica interna;
and form articular knee arterial network? D. a. thoracica lateralis;
A. a. genus media; E. * a. thoracica superior and a. thoracoacromialis;
B. a. genus superior lateralis;
C. a. genus superior medialis; 312. Which of the following muscles takes part in abduction of
D. a. genus inferior lateralis; foot?
E. * all of these;
A. mm. adductor magnus, longus et brevis;
305. Damage of which of the following nerves of lower B. m. gracilis, m. piriformis, m. tensor fasciae latae;
extremity leads to pes equinus? C. m. obturatorius internus, m. quadratus femoris;
A. n. peronaeus superficialis; D. mm. gemelli superioret inferior;
B. n. peronaeus profundus; E. * m. gluteus medius, m. gluteus minimus, m. tensor fasciae
C. n. tibialis; latae;
D. n. saphenus.
E. * n. peronaeus communis;
Clinical Anatomy: An Illustrated Review with Questions and Explanations by Richard Snell 4th Ed.

313. Indicate element of main vasculo-nervous bundle in E. * between m. biceps femoris from outside, and m.
clavipectoral triangle: semimembranosus from inside
A. trunks of brachial plexus;
B. v. basilica; 320. Which of the following nerves is terminal branch of
C. n. axillaris; superficialis peroneal nerve?
D. n. musculocutaneus; A. n. cutaneous intermedius;
E. * a. axillaris; B. n. cutaneous lateralis;
C. n. surae;
314. Which of the following structures of main vasculo-nervous D. n. saphenus;
bundle could be recognised in subpectoral triangle: E. * n. cutaneous medialis;
A. n. medianus;
B. v. basilica; 321. Which of the following muscles attaches to the head of the
C. n. musculocutaneus; fibula?
D. n. ulnaris; A. m. semitendinosus;
E. * v. cephalica; B. m. semimembranosus;
C. m. quadriceps femoris;
315. Where in middle third of brachial region ulnar nerve is D. m. adductor magnus;
located? E. * m. biceps femoris;
A. behind v. cephalica;
B. medial to brachial artery; 322. The medial wall of lacuna vasorum is formed by:
C. outwards from the basilar artery; A. v. femoralis;
D. within sheath of m. biceps brachii; B. lig. surae superior;
E. * between a. brachialis and v. brachialis; C. arcus iliopectineus;
D. lig. pectineale;
316. Where is the n. medianus in the middle third of the brachial E. * lig. lacunare;
region?
A. between m. flexor digitorum superficialis and m. palmaris 323. Bottom of femoral triangle is formed by which of the
longus; following muscles?
B. between m. flexor digitorum profundus and m. palmaris A. m. adductor brevis, m. gracilis;
longus; B. m. adductor longus, m. iliopsoas;
C. between m. flexor pollicis superficialis and m. flexor pollicis C. m. iliopsoas;
longus; D. m. gracilis, m. quadriceps femoris;
D. between m. flexor pollicis profundus and m. palmaris longus; E. * m. pectineus, m. iliopsoas;
E. * between m. flexor digitorum superficialis and m. flexor
digitorum profundus; 324. Which of the following anatomical structures goes out
through inferior foramen of adductor canal?
317. Superior hole of canalis cruropopliteus is bounded by: A. a. et v. femoralis;
A. from behind – m. popliteus, from front – arcus tendineus m. B. a. et v. femoralis, n. saphenous;
solei; C. a. et v. femoralis, n. ishiadicus;
B. from sides – lamina profunda fasciae cruris, from front – m. D. n. femoralis;
popliteus; E. * a., v. et n. femoralis;
C. from inside – m. flexor digitorum longus, from outside – m.
tibialis posterior; 325. The lateral wall of lacuna vasorum is formed by:
D. from front – m. soleus, from behind – n. peroneus profundus; A. a. femoralis;
E. * from front – m. popliteus, from behind – arcus tendineus m. B. lig. lacunare;
solei; C. a. epigastrica inferior;
D. lig. pectineale;
318. To name muscles which starts from tuber ishiadicus: E. * lig. іlio-pectineus;
A. m. semimembranosus;
B. m. semitendinosus; 326. Which of the following anatomical structures forms external
C. m. biceps femoris; wall of femoral canal?
D. none of these; A. a. femoralis;
E. * all of these; B. n. saphenus;
C. n. ishiadicus;
319. Where is the n. ishiadicus in the middle third of the thigh D. n. femoralis;
region? E. * v. femoralis;
A. between m. semimembranosus and m. semitendinosus;
B. between lower edge of m. gluteus maximus and deep fascia; 327. Which of the following ligaments reinforce capsule of hip
C. between m. adductor magnus from front and caput longum of joint?
m. biceps femoris from behind; A. lig. pubofemorale, zona orbicularis;
D. all of these; B. lig. capitis femoris, lig. pubofemorale, lig. ishiofemorale;
C. zona orbicularis, lig. transversum acetabuli;
Clinical Anatomy: An Illustrated Review with Questions and Explanations by Richard Snell 4th Ed.

D. all of these; C. silk continuous;


E. * lig. ishiofemorale, lig. iliofemorale, lig. pubofemorale; D. katgut continuous;
E. * katgut nodular;
328. Indicate which methods of treatment of varix desease do
you know? 336. Which of the following surgical approaches is best to pelvic
A. conservative; part of ureters?
B. sclerosing (injection); A. Kocher’s;
C. surgical; B. Bergman-Israel’s;
D. combined; C. Fedoroff’s;
E. * all of these; D. Pfannenshtil’s;
E. * Pyrogoff’s;
329. Which of the following ligaments reinforce ankle joint from
medial side? 337. To confirm the diagnosis of ectopic pregnancy, which was
A. lig. talofibulare anterius; interrupted, puncture of posterior fornix of vagina was
B. lig. talofibulare posterius; performed. How deep puncture of posterior vaginal vault is
C. lig. calcaneofibulare; injected with a needle?
D. all of these; A. 1 - 2 см;
E. * lig. mediale (deltoideum); B. 4 - 5 см;
C. 0,5 - 1 см;
330. Anterior wall of femoral canal is formed by: D. 5 - 6 см.
A. m. adductor longus; E. * 2 - 3 см;
B. v. femoralis;
C. m. gracilis; 338. Suturing of mucous layer of urinary bladder in case of
D. f. pectinea; cystostomy could lead to the:
E. * f. lata; A. inflammation;
B. bleeding;
331. Superior wall of obturator canal is formed by: C. edema of urinary bladder
A. foramen obturatum; D. fistula
B. membrana obturatoria; E. * formation of stones
C. m. obturatorius internus;
D. m. obturatorius externus; 339. To confirm the diagnosis of nfectious meningitis, spinal
E. * sulcus obturatorius os pubis; puncture was performed. Which of the following layers needle
passes in case of lumbar puncture?
332. Cellular retroperitoneal space is representetad by which of A. lig. supraspinale;
the following layers: B. lig. interspinale;
A. retroperitoneal, perirenal, periureteral cellular spaces; C. lig. flavum with dura mater;
B. retroperitoneal, perirenal; D. skin with subcutaneous tissue
C. perirenal, periureteral, pericystic; E. * all of these;
D. perirenal, pericolic, pericystic
E. * pericolic, perirenal, periureteral, retroperitoneal 340. Subcutaneous paraproctitis should be drained by which of
the following incisions
333. Which of the following vessels and organs are located in
retroperitoneal space? A. from inside of rectum;
A. aorta, kidney, ureters; B. z-shaped incision, 4-5 cm from anus;
B. suprarenal glands, kidney, inferior caval vein; C. through posterior fornix of vagina;
C. kidney, suprarenal gland, ureters; D. through anterior fornix of vagina;
D. kidney, suprarenal gland, renal arteries E. * radial incision to anus;
E. * kidney, suprarenal gland, ureters, abdominal aorta, inferior
caval vein; 341. Submucous paraproctitis should be drained by which of the
following incisions?
334. Which type of laparotomy should be used in case of ectopic A. radial incision to anus;
pregnancy? B. arcuate incision, 4-5 cm from anus;
A. median middle laparotomy; C. through posterior fornix of vagina;
B. transrectal approach; D. z-shaped incision.
C. transversal Pfannenshtil’s approach; E. * from inside of rectum;
D. superior middle laparotomy;
E. * inferior middle laparotomy; 342. Which of the followig incisions is used in case of ishio-
rectal paraproctitis?
335. Which of the following sutures is used in case of A. from inside of rectum;
cystostomy? B. through anterior fornix of vagina;
A. silk nodular; C. through posterior fornix of vagina;
B. kapron nodular; D. radial incision to anus.
Clinical Anatomy: An Illustrated Review with Questions and Explanations by Richard Snell 4th Ed.

E. * arcuate incision, 4-5 cm from anus; A. superior and inferior


B. saggital and frontal
343. Floor of Pti’s triangle is formed by: C. supraampular and ampular
A. m. serratus post. inf. D. peritoneal and infraperitoneal
B. m. obliquus abdominis ext. E. * pelvic and perineal
C. f. lumbodorsalis
D. m. latissimus dorsi 352. Which of the following venous plexus could be recognised
E. * m. obliquus abdominis int. in rectum?
A. external and internal;
344. Leshaft-Hryunfeldt’s rhombus is bounded from above by: B. submucous, subcutaneous
A. ХІІ rib C. superior, middle, inferior
B. medial edge of m. obliquus abdominis ext. D. all are false
C. f. lumbodorsalis E. * submucous, subcutaneous, subfascial
D. all of these
E. * lower edge of m. serratus post. inf. 353. Which of the following positions of uterus in pelvis is
normal?
345. Leshaft-Hryunfeldt’s rhombus is bounded from above by: A. retroversio, retroflexio;
A. ХІІ rib B. anteversio, retroflexio;
B. lower edge of m. serratus post. inf. C. retroversio, anteflexio;
C. m. latissimus dorsi D. curved forward in saggital plane зігнута
D. all are false E. * Anteversio, anteflexio;
E. * m. obliquus abdominis int.
354. We can identify _______ muscular layers in gluteal region.
346. Which of the following anatomical structures are included A. two
in renal peduncle? B. one
A. renal vessels, ureter; C. four
B. renal artery and vein; D. five
C. renal vessels, nerves, ureter; E. * three
D. lymph vessels, ureter;
E. * renal artery and vein, renal pelvis 355. Which of the following muscles create superficial layer of
347. The ureter has ______________ narrowings. gluteal region?
A. one A. m. gluteus med., m. gluteus max
B. two B. m. gluteus max, m. quadratus femoris
C. four C. m. quadratus femoris, . m. gluteus med., m. gluteus max
D. all are false D. m. piriformis, mm. gemelli
E. * three E. * m. gluteus max.

348. Which unpaired branches arise from abdominal aorta? 356. Which of the following muscles create middle layer of
A. a. mesenterica sup., a. renalis, a. lumbalis. gluteal region?
B. a. mesenterica inf., aa. рhrenicae inf. A. m. gluteus max, m. quadratus femoris
C. a renalis, a. lumbalis. B. m. obturatorius int., mm. gemelli, m. quadratus femoris,
D. aa. рhrenicae inf. C. m. quadratus femoris, . m. gluteus med., m. gluteus max
E. * tr. coeliaca, a. mesenterica sup., a. mesenterica inf, a. D. m. obturatorius int., mm. gemelli,
sacralis mediana E. * m. gluteus med., m. piriformis, m. obturatorius int., mm.
gemelli, m. quadratus femoris,
349. Is this necessary to stitch mucous layer in case of suturing of
ureters? 357. Which of the following muscles create deep layer of gluteal
A. yes; region?
B. in according to types of grafting; A. m. obturatorius int., mm. gemelli,
C. in according to types of sutures; B. m. gluteus max, m. quadratus femoris
D. in according to incision C. mm. gemelli, m. quadratus femoris,
E. * no; D. m. gluteus min.
E. * m. gluteus min., m. obturatorius ext.
350. Which of the following recesses is used to confirm of
diagnose of intraabdominal bleeding? 358. A. et v. glutea sup., n. gluteus sup. go out from pelvic cavity
A. recto-vesical through which of the following foramen?
B. retrorectal A. for. infrapiriforme
C. utero-vesical B. great sciatic
D. prevesical C. lesser sciatic
E. * utero-rectal D. external ring of femoral canal
E. * for. suprapiriforme
351. Rectum is divided into two main parts:
Clinical Anatomy: An Illustrated Review with Questions and Explanations by Richard Snell 4th Ed.

359. Internal pudendal artery, inferior gluteal artery and sciatic 367. Describe the direction of Rozer-Nelaton’s line?
nerve go out from pelvic cavity through __________ foramen? A. from superior anterior iliac spine to coccyx;
A. lesser sciatic B. between right and left superior anterior iliac spines;
B. for. suprapiriforme C. from symphysis to tip of coccyx;
C. external ring of femoral canal D. all are false
D. great sciatic E. * from superior anterior iliac spine to sciatic tuber, through
E. * for. infrapiriforme gluteal region;

360. N. ischiadicus goes out from pelvic cavity through which of 368. Greater trochanter of femur bone displaces
the following foramen? ________________________ in case of fractures of neck of
A. great sciatic femur?
B. external ring of femoral canal A. left Rozer-Nelaton’s line;
C. for. suprapiriforme B. B. right Rozer-Nelaton’s line;
D. lesser sciatic C. on Rozer-Nelaton’s line;
E. * for. іnfrapiriforme D. all are false
E. * above Rozer-Nelaton’s line;
361. Scarp’s triangle is bounded from above by:
369. Which of the following anatomical structures is located
A. m. sartorius within lig. capitis femoris ?
B. m. ilio-psoas A. n. obturatoriusB. a. glutea inferior
C. m. pectineus C. n. gluteus superior
D. f. latae D. a. profunda femoris
E. * lig. Inguinale E. * r. acetobularis from a. obturatoria

362. Which of the following muscles creates floor of femoral 370. A patient after the injury diagnosed anterior-medial hip
triangle? dislocation. Between which of the following ligaments hip
A. m. sartorius, m. pectineus. dislocation could be present?
B. m. quadratus femoris, A. ilio-femoral and ischio-femoral;
C. mm. gemelli B. pubo-femoral and ischio-femoral;
D. m. quadratus femoris, m. sartoriusE. * m. ilio-psoas, m. C. pubo-femoral and inguinal;
pectineus. D. ischio-femoral and inguinal;
E. * ilio-femoral and pubo-femoral;
363. Which of the following anatomical structures go through
lacuna vasorum? 371. Between which of the following ligaments anterior-lower
A. a. pudenda int., a. glutea inf., hip dislocation could be diagnosed?
B. a. et v. glutea sup. A. ilio-femoral and pubo-femoral;
C. a. glutea inf., n. pudendus B. pubo-femoral and ischio-femoral;
D. n. ischiadicus, n. cutaneus femoris post. C. pubo-femoral and inguinal;
E. * a. et v. femoralis D. ischio-femoral and inguinal;
E. * pubo-femoral and ischio-femoral;
364. Lacuna vasorum is bounded from outside by:
A. lig. inguinale 372. Which of the following foramens are bounded by lig.
B. m. ilio-psoas sacrospinale and lig. sacrotuberale ?
C. f. latae A. obturator, greater sciatic;
D. a. et v. femoralis B. greater sciatic, suprapiriform;
E. * lig. ilio-pectineum C. suprapiriform, infrapiriform;
D. infrapiriform, greater sciatic;
365. Lacuna vasorum is bounded from inside by: E. * lesser sciatic, greater sciatic;
A. lig. inguinale
B. lig. ilio-pectineum 373. Intermuscular cellular space of ischiadic region connects
C. f. latae with which of the following cellular spaces?
D. a. et v. femoralis A. ischio-rectal fossa;
E. * lig. lacunare (Gimbernati) B. posterior femur bed;
C. popliteal fossa through posterior femur bed;
366. Which of the following anatomical structures go through D. pelvic fat;
lacuna musculorum? E. * all are correct;
A. a. et v. femoralis
B. n. pudendus 374. Which of the following anatomical structures go through
C. n. ischiadicus Hunter’s canal (femoro-popliteal)?
D. n. cutaneus femoris post A. a. et v. circumflexa femoris medialis, n. obturatorius;
E. * m. ilio-psoas et n. femoralis B. a. et v. profunda femoris, n. obturatorius;
C. a., v. et n. femoralis;
Clinical Anatomy: An Illustrated Review with Questions and Explanations by Richard Snell 4th Ed.

D. a. et n. femoralis, greater peroneal vein; 382. Name the superior projection point of the n. ishiadicus that
E. * a. et v. femoralis, saphenous nerve; is used for conduction anesthesia.
A. internal edge of tuber ishiadicus;
375. What is "corona mortis"? B. tip of greater trochanter of femoral bone;
A. femoral artery lies medially to femoral vein; C. point between internal and middle thirds of inguinal ligament;
B. wall of femoral vein is border of internal ring of femoral D. midpoint of inguinal ligament;
canal; E. * midpoint between internal edge of sciatic tuber and tip of
C. inferior epigastric artery goes through lacuna vasorum; greater trochanter of femoral bone;
D. femoral artery gives off main type branches;
E. * obturator artery or its branches lie within lacunar ligament; 383. Between which of the following ligaments hip dislocation
could be diagnosed?
376. A patient with stab wound in middle third of anterior A. ilio-femoral and pubo -femoral;
surface of thigh has bleeding and hematoma. Which of the B. pubo-femoral and ischio-femoral;
following vessels could be damaged in this situation? C. pubo-femoral and inguinal;
A. v. femoralis; D. ischio-femoral and inguinal;
B. a. profunda femoris; E. * ilio-femoral and ischio -femoral;
C. a. obturatoria;
D. all are correct; 384. Which of the following layers should be cutted for approach
E. * a. femoralis; to the sciatic nerve in middle third of posterior surface of thigh?
A. skin, subcutaneous tissue, superficial fascia, f. lata, m.
377. Where the superior projection point of the femoral artery is semitendinous;
located? B. skin, subcutaneous tissue, superficial fascia, f. lata, m.
A. between external and middle thirds of inguinal ligament; semimembranous;
B. between internal and middle thirds of inguinal ligament; C. skin, subcutaneous tissue, superficial fascia, f. lata, mm.
C. midpoint of internal third of inguinal ligament; semitendinous et semimembranous;
D. midpoint of external third of inguinal ligament; D. skin, subcutaneous tissue, superficial fascia, f. lata, m. biceps
E. * midpoint of inguinal ligament; femoris
E. * skin, subcutaneous tissue, superficial and proper fasciae,
378. Which of the following arteries realizes blood suppling of separating muscles;
muscles of posterior bed of thigh?
A. a. circumflexa femoris medialis; 385. Name the most important landmark in the wound to find the
B. a. circumflexa femoris lateralis; superficial femoral ring?
C. a. femoralis; A. v. saphena parva;
D. a. obturatoria; B. v. femoralis;
E. * a. profunda femoris; C. a. femoralis;
D. n. femoralis;
379. Name the inferior projection point of the femoral nerve E. * v. saphena magna;
A. tubercle of medial condyle of femur;
B. medial condyle of femur; 386. A patient with stab wound in lower third of thigh has arterial
C. midpointbetween gibbosity of tibial bone and head of fibular bleeding. In case of ligation of femoral artery within femoral
bone; triangle m. sartorius should be displaced:
D. 1 сm backward from internal edge of tibial bone; A. inside;
E. * middle of popliteal fossa; B. forward;
C. backward;
380. Where is place for best ligation of femoral artery (to D. upward;
preserve the collateral blood flow)? E. * outside;

A. below arising of a. profunda femoris; 387. Which of the following layers we should cut in case of
B. below arising of aa. genus superioris; denudation of femoral artery within femoral triangle?
C. below arising of aa. genus inferioris; A. skin, subcutaneous tissue, superficial fascia;
D. below arising of a. genus descendens; B. skin, subcutaneous tissue, superficial fascia, proper fasciae, m.
E. * above arising of a. profunda femoris; sartorius;
C. skin, subcutaneous tissue, superficial fascia, proper fasciae, m.
381. From which side and with which of the following semimembranous;
instruments we should put ligature on femoral artery after injury D. skin, subcutaneous tissue, superficial fascia, proper fasciae, m.
of this vessel in lower third of thigh? semitendinous;
A. from internal wall of artery with sharp Deshan’s needle; E. * skin, subcutaneous tissue, superficial fascia, proper fasciae;
B. from external wall of artery with blunt Deshan’s needle;
C. from external wall of artery with sharp Deshan’s needle; 388. Name the inferior projection point of the femoral artery
D. from external wall of artery with straight Billet’s forceps; A. lateral condyle of femur;
E. * from internal wall of artery with blunt Deshan’s needle; B. tip of popliteal rhombus;
Clinical Anatomy: An Illustrated Review with Questions and Explanations by Richard Snell 4th Ed.

C. midpoint between gibbosity of tibial bone and head of fibular


bone; 396. Where is place for best ligation of popliteal artery (to
D. 1 сm backward from internal edge of tibial bone; preserve the collateral blood flow)?
E. * tubercle of medial condyle of femur; A. above arising of a. genus superior lateralis;
B. below arising of a. genus superior medialis and a. genus
389. Which of the following nerves may radiate pain in the knee superior;
joint at adnexitis? C. below arising of a. genus superior lateralis;
A. n. femoralis; D. above arising of a. genus descendens;
B. motor branches of n. obturatorius; E. * above arising of a. genus superior medialis;
C. n. ishiadicus;
D. n. pudendus; 397. Which of the following surgical instruments surgeon needs
E. * cutaneous branche of n. obturatorius; for ligation of popliteal artery and from which side he should put
ligature on vessel?
390. Ken’s line is projection line for which of the following A. sharp Deshan’s needle, from outside;
arteries? B. straight Bilrot’s forceps, from outside;
A. a. brachialis; C. blunt Deshan’s needle, from inside;
B. a. subclavia; D. curved Bilrot’s forceps, from outside;
C. a. axillaris; E. * blunt Deshan’s needle, from outside;
D. a. tibialis anterior;
E. * a. femoralis; 398. A surgeon performes catheterization of popliteal artery.
Which of the following surgical approaches is best for saving
391. Hritti-Szymanowskyy’s amputation of thigh in lower third neurovascular components?
belongs to which of the following types of amputations? A. through middle of popliteal fossa;
A. fascioplastic; B. long diagonal of popliteal rhombus, something inside;
B. conico-circular; C. long diagonal of popliteal rhombus, something outside;
C. myoplastic; D. anterior-external side of thigh;
D. circular; E. * anterior-internal side of thigh;
E. * osteoplastic;
399. Name the external landmarks for the projection line of
392. A patient with traumatic compression of sciatic nerve. common peroneal nerve in the popliteal fossa?
Which of the following groups of muscles suffer in case of injury A. m. quadriceps femoris, m. semimembranous, m.
of n. ishiadicus? semitendinous, external surface of neck of fibula;
A. anterior group of thigh, anterior and posterior groups of leg; B. m. biceps femoris, medial condyle of femur;
B. posterior group of thigh, anterior and posterior groups of leg; C. m. biceps femoris, lateral condyle of femur;
C. posterior group of thigh, anterior and external groups of leg; D. m. semimembranous, m. semitendinous, , lateral condyle of
D. anterior group of thigh, posterior and external groups of leg; femur;
E. * posterior group of thigh, anterior, posterior and external E. * m. biceps femoris, m. semimembranous, m. semitendinous,
groups of leg; external surface of neck of fibula;

393. Which of the following anatomical structures has name 400. A patient diagnosed with hemarthrosis of the knee joint.
“adductor canal”? Which artery is its source?
A. canalis cruro-popliteus; A. а. genus superior lateralis;
B. canalis spiralis; B. а. genus superior medialis;
C. canalis musculo-peronealis; C. а. genus inferior medialis;
D. canalis pudendalis. D. а. genus inferior lateralis;
E. * canalis femoro-popliteus; E. * а. genus media;

394. Which way to detect popliteal artery provides a wide 401. A. tibialis posterior and n. tibialis go through which of the
surgical access? following canal?
A. long diagonal of popliteal rhombus, something outside; A. canalis malleolaris;
B. anterior-internal side of thigh; B. canalis musculo-peronealis superior;
C. anterior-external side of thigh; C. canalis musculo-peronealis inferior;
D. through popliteal fossa in transverse direction; D. canalis femоro-popliteus.
E. * long diagonal of popliteal rhombus, something inside; E. * canalis cruro-popliteus;

395. Fat of popliteal fossa connects with which of the following 402. A patient with stab wound in region of popliteal fossa,
cellular spaces? bleeding. Which of the following vasculo-nervous structures of
A. posterior bed of thigh; popliteal fossa is located most deeply?
B. posterior bed of leg; A. v. poplitea:
C. anterior bed of leg; B. n. tibialis;
D. fat of gluteal region through posterior bed of thigh; C. v. saphena parva;
E. * all of these; D. n. tibialis communis;
Clinical Anatomy: An Illustrated Review with Questions and Explanations by Richard Snell 4th Ed.

E. * a. poplitea; E. * from midpoint between malleoli to I interosseal metatarsal


space;
403. A patient with phlegmon of posterior surface of thigh,
inflammation spread to the popliteal fossa. In which of the 410. A patient with anaerobic infection of thigh wound.
following directions inflammations from deep popliteal cellular Guillotine amputation was performed. This surgery belongs to
space could be spread to? which of the following types of amputations?
A. obturator canal, anterior muscular bed of thigh and leg; A. conico-circular;
B. adductor canal, anterior muscular bed of leg; B. single-flap;
C. obturator canal, external muscular bed of leg; C. with cuff;
D. adductor canal, internal muscular bed of thigh; D. double-flap;
E. * adductor canal, posterior muscular bed of thigh and leg; E. * circular;

404. To arrest bleeding а. tibialis роsterior was ligated in middle 411. A flap in case of fascioplastic method of amputation
third of leg. Which of the following arteries is main source of consists of which of the following layers?
blood suppling of lateral bed of leg in case of ligation а. tibialis A. skin, subcutaneous tissue, superficial fascia;
роsterior in middle third of leg? B. skin, subcutaneous tissue;
A. а. tibialis роsterior; C. skin, subcutaneous tissue, superficial fascia and periosteum;
B. a. dorsalis pedis; D. skin, subcutaneous tissue, superficial fascia, tendons.
C. а. tibialis anterior; E. * skin, subcutaneous tissue, superficial and proper fasciae;
D. none of these;
E. * a. peronea; 412. Foramen trilaterum connects which of the following spaces?
Які простори з’єнуються між собою за допомогою?
405. A surgeon performes approach to а.t ibialis роsterior in A. cavity of the shoulder joint and axillary cavity;
middle third of leg. Cruro-popliteal canal is bounded by which of B. spatium supraspinatum and infraspinatum;
the following muscles? C. spatium supraspinatum and spatium subscapularis;
A. m. flexor digitorum longus; D. all are false;
B. m. flexor hallucis longus; E. * scapular region and axillary cavity;
C. m. soleus;
D. m. tibialis posterior; 413. Where is place of injection in case of puncture of the
E. * all of these; shoulder joint on external surface of shoulder?
A. on 1 сm lateral from greater tubercle of humeral bone;
406. A man with trauma of leg Amputation was performed in 10 B. above acromion;
hours after injury. It was: C. beneath posterior edge of acromion;
A. secondary late; D. along posterior edge of m. deltoideus;
B. secondary early; E. * on 2 сm below acromion, above greater tubercle of humeral
C. reamputation; bone;
D. necrectomy.
E. * primary; 414. A. circumflexa scapulae and deep branches of a. transversa
colli are located under which of the following muscles?
407. Where is projection and palpation point of a. dorsalis pedis? A. m. serratus inferior;
A. in front of lateral melleolus; B. m. teres minor;
B. in projection of I metatarsal bone; C. m. infraspinatus.
C. in projection of II cuneiform bone; D. m. deltoideus;
D. none of these; E. * m. trapezius;
E. * in front of medial melleolus;
415. Which of the following vasculo-nervous bundles lies in
408. A patient has no pulse on а. tibialis роsterior. Where is superficial subpectoral space?
projection and palpation point of a. tibialis роsterior? A. anterior branches of thoracic nerves;
A. lateral surface of lateral melleolus; B. internal thoracic artery;
B. near superior edge of Achilles tendon; C. brachial plexus;
C. near internal edge of Achilles tendon; D. subcostal nerve;
D. medial surface of medial melleolus; E. * thoraco-acromial artery;
E. * posterio-medial surface of medial melleolus;
416. Which of the following veins goes through subpectoral
409. Where is projection line of a. dorsalis pedis? space, penetrate the deep fascia and fall into subclavian vein?
A. from tuberositas tibiae to medial melleolus; A. brachiocephalic vein;
B. from tuberositas tibiae to midpoint between medial melleolus B. deep cervical vein;
and Achilles tendon; C. lateral cutaneous vein;
C. 1 сm from medial edge of tibial bone to midpoint between D. anterion intercostal vein;
medial melleolus and Achilles tendon; E. * basal vein;
D. from middle of popliteal fossa downwards vertically;
Clinical Anatomy: An Illustrated Review with Questions and Explanations by Richard Snell 4th Ed.

417. Which of the followig structures of main vasculo-nervous D. n. thoracospinalis, a. thoracica superior;
bundle of axillary fossa is located most superficial and medial? E. * n. thoracicus longus, a. thoracica lateralis;
A. n. medianus;
B. n. ulnaris; 425. A. collateralis ulnaris superior goes together with which of
C. n. radialis; the following nerves?
D. n. musculo-cutaneus; A. n. medianus;
E. * v. axillaris; B. n. radialis;
C. n. humerus posterior
418. Which of the followig structures of main vasculo-nervous D. n. musculo-cutaneus;
bundle of axillary fossa is located behind the axillary artery? E. * n. ulnaris;
A. n. ulnaris;
B. n. radialis; 426. Indicate the greatest danger of wounds of gluteal region:
C. n. cutaneus antebrachii; A. possibility of paraproctitis;
D. n. cutaneus brachii; B. possibility of abscess;
E. * n. medianus; C. possibility of phlegmon;
D. bleeding from wound
419. Which of the following structures is located in front of E. * bleeding into the pelvic fat;
subscapular artery in axillary fossa?
A. n. radialis; 427. Main vasculo-nervous bundles of gluteal region are located:
B. n. ulnaris; A. in subcutaneous tissue;
C. n. axillaris; B. under middle muscular layer;
D. a. axillaris; C. under m. gluteus minimus;
E. * n. medianus; D. under fascia;
E. * under m. gluteus maximus;
420. Which of the following nerves arises from posterior trunk of 428. Medial wall of femoral canal is represented by:
brachial plexus? A. femoral artery;B. femoral nerve;
C. * lacunar ligament;
A. n. medianus; D. iliopsoas muscle;
B. n. ulnaris; E. femoral vein;
C. n. musculo-cutaneus;
D. n. cutaneus brachii; 429. Anterior wall of femoral canal is represented by:
E. * n. radialis; A. femoral artery;
B. femoral nerve;
421. The biggest nerve of axillary fossa is: C. lacunar ligament;
A. n. medianus; D. fascial sheath of femoral vein;
B. n. ulnaris; E. * superficial layer of f. lata;
C. n. cutaneus brachii;
D. n. musculo-cutaneus; 430. Femoral nerve goes:
E. * n. radialis. A. in lacuna vasorum;
B. between femoral artery and vein;
422. Which of the following arteries arise from axillary artery C. in lacunar ligament
near superior edge of tendon of m. latissimus dorsi? D. in adductor muscles;
A. a. thoracoacromialis; E. * in lacuna musculorum;
B. a. subscapularis;
C. a. circumflexa humeri posterior and a. thoracicasuperior; 431. In which region of arm projection point of n. cutaneus
D. a. circumflexa scapulae; brachii superior lateralis is located (in the place of output in the
E. * a. circumflexa humeri posterior and a. circumflexa humeri subcutaneous tissue)?
anterior; A. middle third;
B. lower third;
423. Collateral circulation after ligation of axillary artery is C. midpart
realized through which of the following arteries? D. between middle and lower thirds;
A. a. thoracica superior; E. * superior third;
B. a. thoracoacromialis;
C. a. circumflexa scapulae; 432. Where is projection point of axillary nerve on posterior
D. a. circumflexa humeri posterior; surface of humerus (in according to acromion)?
E. * a. subscapularis; A. 4 сm apart;
B. 5 сm apart;
424. Which of the following vasculo-nervous bundles is located C. 6 сm apart;
on medial wall of axillary cavity? D. 7 сm apart;
A. n. cutaneus lateralis, a. thoracospinalis; E. * 2 сm apart;
B. n. intercosto-brachialis, a. subscapularis;
C. a. thoracica superior, n. subcostalis;
Clinical Anatomy: An Illustrated Review with Questions and Explanations by Richard Snell 4th Ed.

433. Which of the following vasculo-nervous bundles is located C. backwards;


in subdeltoid cellular space? D. individually;
A. n. radialis, a. circumflexa humeri posterior; E. * laterally;
B. n. musculo-cutaneus, a. circumflexa humeri anterior;
C. a. circumflexa humeri posterior, n. ulnaris; 441. A. brachialis profunda divides into which of the following
D. n. subscapularis, a. circumflexa humeri anterior; arteries?
E. * n. axillaris, a. circumflexa humeri posterior;
A. arteries to humeral bone;
434. From which of the following nervous structures B. r. deltoideus;
musculocutaneous nerve arises? C. a. collateralis media;
A. medial trunk of brachial plexus; D. a. collateralis radialis;
B. posterior trunk of brachial plexus; E. * all of these;
C. n. medianus;
D. n. ulnaris; 442. Which of the following nerves is the most frequently
E. * lateral trunk of brachial plexus; damaged nerve in fractures of the surgical neck of humerus?
A. n. medianus;
435. N. cutaneus brachii posterior arises from which of the B. n. radialis;
following nervous structures? C. n. ulnaris;
A. medial trunk of brachial plexus; D. n. musculo-cutaneus;
B. posterior trunk of brachial plexus; E. * n. axillaris;
C. n. musculo-cutaneus;
D. n. ulnaris; 443. Which vein is not accompanied by cutaneous nerve in the
E. * lateral trunk of brachial plexus; cubital fossa?

436. In case of fractures of the humerus which of the following A. v. cephalica;


nerves could be most frequently damaged? B. both are accompanied;
A. n. medianus; C. individually;
B. n. ulnaris; D. both are not accompanied;
C. n. musculo-cutaneus; E. * v. basilica;
D. n. axillaris;
E. * n. radialis; 444. Radial nerve divides into deep and superficial branches at
which of the following levels?
437. What nerve is adjacent to the shoulder joint capsule? A. 5-7 сm above processus styloideus of ulnar bone;
B. 5-7 сm below processus styloideus of ulnar bone;
A. n. radialis; C. at level of processus styloideus of ulnar bone;
B. n. ulnaris; D. 2 сm above of joint line;
C. n. thoraco-spinalis; E. * at level of joint line;
D. n. intercosto-brachialis;
E. * n. axillaris; 445. Which of the following vessels should be ligated in case of
injury of gluteal region?
438. From which of the following nervous structures n. cutaneus A. a. glutea superior;
brachii lateralis superior arises? B. a. pudenda interna;
C. a. obturatoria;
A. lateral trunk of brachial plexus; D. a. iliaca externa;
B. medial trunk of brachial plexus; E. * a. iliaca interna;
C. posterior trunk of brachial plexus;
D. n. axillaris; 446. Which of the following arteries is the biggest branch of
E. * n. musculo-cutaneus; femoral artery?
A. a. epiusgastrica superficialis;
439. Which of the following superficial veins is accompanied by B. a. epigastrica superficialis;
n. cutaneus brachii? C. a. pudenda externa;
D. a. genus descendens;
A. v. cephalica; E. * a. profunda femoris;
B. v. mediana;
C. v. ulnaris; 447. Which of the cutaneous branches of femoral nerve lies most
D. v. radialis; medial?
E. * v. basilica; A. n. cutaneus femoris lateralis;
B. anterior cutaneous branches;
440. Median nerve is placed ______________ in relation to the C. ilio-inguinal nerve;
brachial artery. D. individually;
A. medially; E. * saphenous nerve;
B. forwards;
Clinical Anatomy: An Illustrated Review with Questions and Explanations by Richard Snell 4th Ed.

448. Sciatic nerve divides into its branches at which of the


following distance from joint line? 456. A. рoplitea goes in which of the following canals after
A. 10-12 сm; leaving from popliteal fossa?
B. 13-14 сm; A. сanalis adductorius;
C. 14-15 сm; B. canalis musculoperonaeus superior;
D. at level of popliteal fossa; C. canalis musculoperonaeus inferior;
E. * 7-9 сm; D. all are false;
E. * canalis cruropopliteus;
449. Which of the following annatomical sructures goes through
carpal canal? 457. Which of the following anatomical structures takes part in
A. n. ulnaris; forming of walls of canalis cruropopliteus?
B. n. radialis A. m. tibialis posterior;
C. n. medianus; B. lamina profunda fasciae cruris;
D. a. interossea palmaris; C. m. flexor digitorum longus;
E. * tendons of mm. flexor digitorum superficialis et profundus; D. m. soleus;
E. * all of these;
450. Which of the following arteries are branches of deep palmar
arch? 458. Which of the following anatomical structures form anterior
A. aa. digitales palmares communes; wall of canalis cruropopliteus?
B. a. princeps pollicis; A. m. flexor digitorum longus, m. soleus ;
C. a. radialis indicis; B. m. soleus, lamina profunda fasciae cruris;
D. aa. digitales dorsales; C. lamina profunda fasciae cruris;
E. * aa. metacarpeae palmares; D. lamina profunda fasciae cruris, m. tibialis posterior .
E. * m. tibialis posterior, m. flexor digitorum longus;
451. Which of the following arteries could be palpated near
lateral edge of os pisiformae: 459. Which of the following anatomical structures goes in fissure
A. a. radialis; of superficial lamina of leg fascia in middle third of posterior
B. ramus carpeus palmaris; surface of leg?
C. ramus palmaris profundus; A. n. peroneus profundus;
D. a. princeps pollicis. B. v. saphena magna;
E. * a. ulnaris C. v. saphena parva;
D. n. saphenus;
452. Indicate projection of n. peroneus communis in popliteal E. * n. peroneus superficialis;
fossa?
A. lateral edge of m. biceps femoris 460. “Corona mortis” is formed by which of the following
B. medial edge of m. semimembranosus; anatomical structures?
C. head of fibular bone; A. v.femoralis, a. femoralis, a. epigastrica inf.;
D. lateral edge of m. semitendinosus; B. v.femoralis, a. femoralis;
E. * medial edge of m. biceps femoris; C. m. adductor magnus, v. femoralis;
D. lig. inguinale, a. epigastrica inf., a. obturatoria;
453. Bottom of popliteal fossa is formed by which of the E. * v.femoralis, a. epigastrica inf., a. obturatoria;
following anatomical structures?
A. popliteal surface of femur and lig. popliteum obliquum; 461. Which of the following arteries arises from femoral artery
B. m. popliteus; within femoral triangle?
C. femur bone and m. popliteus; A. a. epigastrica superficialis;
D. os tibia and m. popliteus; B. a. circumflexa ilium superficialis;
E. * lig. popliteum obliquum and m. popliteus; C. a. pudenda externa;
D. a. genus descendens;
454. Which of the following nerves are located in popliteal E. * a. profunda femoris;
fossa?
A. n. femoralis; 462. In case of arthrotomy of hip joint which of the following
B. n. cutaneus femoris lateralis; muscles should be cutted?
C. n. ishiadicus, n. peroneus communis; A. m. gracilis;
D. n. peroneus communis; B. m. rectus femoris;
E. * n. tibialis, n. peroneus communis; C. m. pectineus;
455. A. рoplitea enters the popliteal fossa after which of the D. m. iliopsoas;
following canals? E. * m. tensor fasciae latae;
A. canalis cruropopliteus;
B. canalis musculoperonaeus superior; 463. Which of the following surgical approaches is used in case
C. canalis musculoperonaeus inferior; of arthrotomy of knee joint?
D. all are false; A. incision above tendon of m. biceps femoris;
E. * сanalis adductorius; B. incision through popliteal fossa;
Clinical Anatomy: An Illustrated Review with Questions and Explanations by Richard Snell 4th Ed.

C. incision between m. semimembranosus and m. 471. What are the main demand for stitching of vessels?
semitendinosus; A. connection of vessels should be performed with minimal
D. incision through Zhober fossa; narrowing of at the junction;
E. * parapatellar incisions; B. ends of vessels should be connected end-to-end;
C. hermetic anastomoses;
464. Which of the following surgical approaches is used in case D. prevention of blood clots;
of arthrotomy of ankle joint? E. * ends of vessels should be connected by their internal
A. anterior; surfaces;
B. posterior;
C. medial; 472. What are the absolute indications for amputations?
D. median; A. traumatic limb abruption;
E. * lateral; B. chronic osteomyelitis of limb with danger of
development of internal organs amyloidosis;
465. Indicate extra-articular ligaments of knee joint: C. severe purulent infection that threatens the health of the
A. lig. patellae propria; patient (gangrene);
B. lig. collaterale (medial, lateral); D. malignant tumor;
C. lig. popliteum obliguum; E. * all of these;
D. lig. popliteum arcuatum;
E. * all of these; 473. Which of the following incisions is used in case of
subdeltoid phlegmon?
466. Which of the following surgical approaches is used in case A. on posterior edge of m. deltoideus;
of arthrotomy of hip joint? B. through m. deltoideus closer to its anterior edge;
A. posterior; C. on sulcus deltoideo-pectoralis;
B. lateral; D. through middle of m. deltoideus;
C. medial; E. * on anterior edge of m. deltoideus;
D. on the medial edge of the anterior tibial muscle;
E. * combined; 474. Indicate intra-articular ligaments of knee joint:
A. lig. cruciatum anterius;
467. Where surgeon should cut in case of phlegmon of anterior B. lig. cruciatum posterius;
leg region? C. lig. meniscofemorale posterius;
A. on projection line of a. tibialis anterior, departing at 2 cm to D. lig. transversum genus;
the middle; E. * all of these;
B. on projection line of a. tibialis anterior, departing at 2 cm to
the outside; 475. Branches of which of the following nerves take part in
C. on medial edge of m. tibialis anterior; innervation of radiocarpal joint?
D. on projection line of a. tibialis anterior; A. median, radial;
E. * on anterior edge of tibial bone; B. radial, musculo-cutaneous;
C. musculo-cutaneous;
468. Which of the following incisions is used in case of foot D. axillary;
phlegmon? E. * radial, median, ulnar;
A. medial incision on the edge of the foot lifting;
B. lateral incision on the edge of the foot lifting; 476. Which of the following vessels should be ligated in case of
C. medial incision; injury of gluteal region?
D. incision on heel bone and phalanges; A. superior gluteal artery;
E. * incision on lateral and medial edges of aponeurosis plantaris; B. obturator artery;
C. common iliac artery;
469. Which of the following surgical approaches is used in case D. external iliac artery;
of arthrotomy of shoulder joint after Langenbeck? E. * internal iliac artery;
A. 8-10 cm downwards from acromion;
B. on posterior edge of deltoid muscle; 477. At which of the following levels ureters inflow into urinary
C. through deltoid muscle; bladder?
D. on sulcus deltoideo-pectoralis; A. neck
E. * on anterior edge of deltoid muscle; B. body
C. fundus
470. Indicate place for performing of puncture of shoulder joint: D. apex
A. under processus coracoideus; E. * lower wall
B. in space between posterior edge of deltoid muscle and edge of
m. supraspinatus; 478. Which of the following vessels realises arterial supplying of
C. under posterior edge of acromion; superior parts of ureters?
D. above acromion; A. superior rectal artery;
E. * lateral to processus coracoideus; B. abdominal aorta;
C. uterine arteries;
Clinical Anatomy: An Illustrated Review with Questions and Explanations by Richard Snell 4th Ed.

D. obturator artery;
E. * renal artery;

479. Which of the following anatomical structures goes through


urogenital diaphragm in male?
A. spongy part of urethra
B. anal canal
C. lower fascia of pelvic diaphragm
D. testicular arteries
E. * membranous part of urethra

480. Which of the following anatomical structures goes through


urogenital diaphragm in female?
A. anal canal
B. dorsal vein of clitoris
C. dorsal artery of clitoris
D. all are false
E. * urethra, vagina

You might also like